Vous êtes sur la page 1sur 81

INDEX

01-01-2019 ........................................................................................................................................................... 3

02-01-2019 ........................................................................................................................................................... 6

03-01-2019 ........................................................................................................................................................... 9

04-01-2019 ......................................................................................................................................................... 12

05-01-2019 ......................................................................................................................................................... 15

07-01-2019 ......................................................................................................................................................... 18

08-01-2019 ........................................................................................................................................................ 20

09-01-2019 ......................................................................................................................................................... 23

10-01-2019 ......................................................................................................................................................... 27

11-01-2019 ..........................................................................................................................................................29

12-01-2019.......................................................................................................................................................... 33

14-01-2019 ......................................................................................................................................................... 36

16-01-2019 .........................................................................................................................................................38

17-01-2019 .......................................................................................................................................................... 41

18-01-2019 ......................................................................................................................................................... 45

19-01-2019 ........................................................................................................................................................ 48

21-01-2019.......................................................................................................................................................... 51

22-01-2019 ......................................................................................................................................................... 53

23-01-2019 ......................................................................................................................................................... 56

24-01-2019 ......................................................................................................................................................... 59

25-01-2019 ......................................................................................................................................................... 61

26-01-2019 .........................................................................................................................................................64

28-01-2019 ......................................................................................................................................................... 67

29-01-2019 ......................................................................................................................................................... 70

30-01-2019 ......................................................................................................................................................... 74

31-01-2019.......................................................................................................................................................... 78

www.shankariasacademy.com | www.iasparliament.com
01-01-2019 b. 2 only
1) Consider the following statements with c. Both 1 and 2
respect to Photo-DNA Technology d. Neither 1 nor 2
1. It is a technology developed and owned
by Google.
4) Consider the following statements with
2. It is used exclusively to identify child respect to Ujjwala Sanitary Napkins initiative
exploitation images and should not be
used for any other purposes. 1. It is an initiative of Odisha government.
Which of the statement(s) given above is/are 2. The funding for the scheme will be
correct? shared equally by the central and state
government.
a. 1 only
Which of the statement(s) given above is/are
b. 2 only correct?
c. Both 1 and 2 a. 1 only
d. Neither 1 nor 2 b. 2 only
c. Both 1 and 2
2) Consider the following statements with d. Neither 1 nor 2
respect to Armed Forces Special Powers
Act (AFSPA)
1. Under AFSPA, security forces have the 5) ―B V R Mohan Reddy Committee‖ was
powers to conduct operations anywhere sometimes seen in the news recently, which is
and arrest anyone without any prior associated with which of the following?
notice. a. To suggest a roadmap to completely eliminate
2. AFSPA is being effective in the whole of ragging in Indian colleges
Nagaland. b. To suggest a plan for expansion in engineering
Which of the statement(s) given above is/are education
correct? c. To suggest a roadmap to close the skill gap in
a. 1 only India
b. 2 only d. To protect the wetlands in the densely
populated urban areas
c. Both 1 and 2
d. Neither 1 nor 2
6) Consider the following statements with
respect to Bhima Koregaon Battle
3) Consider the following statements 1. It was fought between Peshwa Sambhaji
1. In India, Ancient Monument means any and the English East India Company
structure which is of archaeological or (EIC).
artistic interest and has been in 2. Mahars, who were once part of Maratha
existence for not less than 1000 years. army, fought against Marathas in this
2. In 2016 and 2017, no new monuments battle.
were included in the list of sites of Which of the statement(s) given above is/are
monuments of national importance. correct?
Which of the statement(s) given above is/are a. 1 only
correct?
b. 2 only
a. 1 only
c. Both 1 and 2

www.shankariasacademy.com | www.iasparliament.com
d. Neither 1 nor 2 b. 2 only
c. Both 1 and 2
7) ―Velakali‖ is a traditional martial dance d. Neither 1 nor 2
form performed in which of the following
states?
a. Tamilnadu Answers
b. Kerala 1. b

c. Karnataka  PhotoDNA is a technology developed


and owned by Microsoft.
d. Andhra Pradesh
 It is mainly used to prevent child
pornography from being uploaded on the
8) In India, monuments will be declared as web and is free to use.
of ―National Importance‖ by which of the  The company has restricted the use of
following agencies? this software beyond this purpose in all
a. National Monuments Authority countries.
b. National Commission for Cultural and  The software creates a unique digital signature
Heritage Sites of any image and then compares it to other
photos to find a similar one.
c. Archaeological Survey of India
 Recently, the Central Bureau of Investigation
d. Concerned State governments
(CBI) sent an official request to social media
platforms to use Microsoft's PhotoDNA
for purposes of investigation in regular
9) Consider the following statements with
criminal cases.
respect to Rhesus Macaque
2. c
1. It is endemic to Eastern Himalayas.
2. It has been listed under critically  The Centre had recently extended the Armed
Forces Special Powers Act in Nagaland for six
endangered category of IUCN Red list.
more months, contending that the whole
Which of the statement(s) given above is/are state is in such ―disturbed and dangerous
correct? condition‖ that the use of armed forces in aid
of civil power is necessary.
a. 1 only
b. 2 only  Under AFSPA, security forces have the
powers to conduct operations
c. Both 1 and 2 anywhere and arrest anyone without
any prior notice.
d. Neither 1 nor 2
3. b

10) Consider the following statements with  According to the Ancient Monuments and
respect to Rheum Archaeological Sites and Remains Act, 1958,
an ―Ancient Monument means any structure,
1. It is a thin, watery mucus that is erection or monument, or any tumulus or
discharged from the nose, eyes or place of interment, or any cave, rock-sculpture,
mouth. inscription or monolith which is of historical,
archaeological or artistic interest and which
2. Rheum discharged from the mouth is
has been in existence for not less than 100
called „gound‟.
years."
Which of the statement(s) given above is/are
correct?  Monuments are declared protected and of
national importance by the Archaeological
a. 1 only Survey of India (ASI).

www.shankariasacademy.com | www.iasparliament.com
 In 2016 and 2017, no new monument 600 of his soldiers after which he withdrew
was included in the list of sites of national and gave up plans to attack Pune.
importance.
 British constructed a tower to commemorate
 The last monument to be included in the list, this victory.
in 2015, was the Vishnu Temple in Nadavayal
7. b
in Kerala's Wayanad district.
 Velakali a stylised, martial dance
 In 2018, ASI declared 6 monuments to be of
of Kerala blends movements and postures of
national importance.
Kathakali and Kalaripayattu was originated in
4. d Ambalappuzha.
 The Ujjwala Sanitary Napkins initiative has 8. c
been launched by three oil marketing
 The Ancient Monuments and Archaeological
companies (OMCs) – IOCL, BPCL and
Sites and Remains (AMASR) Act, 1958
HPCL.
authorised the Archaeological Survey of
 The mission, which forms part of the India (ASI) to declare a monument to be of
Corporate Social Responsibility (CSR) national importance by issuing a notification
initiative of OMCs in Odisha. in the Gazette of India.

 The three companies will set up 100  Thereafter, activities relating to preservation
manufacturing units at the Common Service and conservation of monuments were to be
Centres (CSCs). undertaken.
5. b  National Monument Authority (NMA) does
not have any provision to declare
 With more than half the engineering seats monument/site as of national importance.
falling vacant every year, a government
committee, headed by IIT-Hyderabad 9. d
chairman B V R Mohan Reddy, has advised
 It has been found in Afghanistan,
the All India Council for Technical Education
Bangladesh, Bhutan, China, India, Lao
(AICTE) to stop setting up new colleges
People's Democratic Republic,
from 2020 and review the creation of new
Myanmar, Nepal, Pakistan, Thailand, Viet
capacity every two years after that.
Nam.
 The committee was asked to come up with a
 It has been listed under Least
medium and short-term perspective plan
Concern Category of IUCN Red list.
for expansion in engineering education.
6. b
10. a
 It was the last of the Anglo-Maratha
battle that took place on 1 January 1818 in  Rheum is a thin, watery mucus that is
Bhima, Koregaon between the troops of discharged from the nose, eyes or mouth.
Maratha Ruler Baji Rao Peshwa II and
the British East India Company (EIC).  Rheum discharged from the eyes is called
‗gound‘.
 The Mahars, though untouchables, had been
valued for their military skills for centuries  Gound has a very important job — it gathers
and formed a significant portion of foreign particles like dust, pollen, blood cells,
Shivaji‘s army. skin cells, etc., which have entered the eye and
escorts them out.
 However under the Peshwas they were ill-
treated and lost their military glory.  When a person is awake, blinking causes the
gound to be carried away with the moisture
 In the battle, the EIC represented by produced by the tear glands in the eyes.
majority of Mahar soldiers successfully
resisted Peshwa troops in which Peshwa lost

www.shankariasacademy.com | www.iasparliament.com
 However, in the absence of blinking during 3. It may lead to a surge of Indian exports
sleep, gound dries up and accumulates in the into China with very limited access for
corners of the eyes, forming ‗crusties‘. Chinese exports to the Indian market.
 ‗Crusties‘, ‗sleep boogers‘ or ‗sandman‘s dust‘ 4. Facilitate India‟s MSMEs to effectively
are the many names given to the dry, flakey integrate into the regional value and
matter that we find in the corners of our eyes supply chains.
when we wake up in the morning. Select the correct answer using the code given
 They are the dried-up form of what is below
scientifically known as ‗rheum‘. a. 1 only
b. 1 and 2 only
02-01-2019 c. 1, 2 and 4 only
1) Consider the following statements d. 1, 2, 3 and 4
1. Rajasthan was the first State in the
country to fix a minimum educational 4) Consider the following statements with
qualification for contesting elections to respect to UNESCO
the Panchayati Raj Institutions.
1. It was co-founded by the U.S after
2. The Rajasthan government has recently World War II to foster peace.
abolished the condition of a minimum
educational qualification to contest 2. Its headquarters is located in Geneva,
local body elections. Switzerland.
Which of the statement(s) given above is/are 3. Recently, the US and Israel have
correct? officially quit the UNESCO.
a. 1 only Which of the statements given above are
correct?
b. 2 only
a. 1 and 2 only
c. Both 1 and 2
b. 1 and 3 only
d. Neither 1 nor 2
c. 1, 2 and 3
d. None
2) ―Samwad with Students‖ is an initiative
launched recently by which of the following
organisations?
5) ―Madhav Gadgil Committee‖ was sometimes
a. Indian Space Research Organisation (ISRO) seen in the news recently, was associated with
which of the following?
b. Prime Minister‘s Office (PMO)
a. Eastern Himalayas
c. Indian Army
b. Western Ghats
d. Ministry of Environment, Forest and Climate
Change c. Aravalli Mountain
d. Coastal Zone Regulation
3) What is/are the most likely benefits of
Regional Comprehensive Economic
6) "Agreement on the Prohibition of Attack
Cooperation (RCEP) Agreement for India?
against Nuclear installations" was sometimes
1. Provide market access for India‟s goods seen in the news recently. It is an agreement
and services exports. signed between which of the following two
countries?
2. Encourage greater investments and
technology into India. a. Israel and Palestine

www.shankariasacademy.com | www.iasparliament.com
b. North Korea and South Korea a. A special technique used during periods of
high inflation whereby financial statements are
c. India and Pakistan
adjusted according to price indexes
d. USA and Cuba
b. An inflationary situation, where wages presses
prices up and prices pulls wages up
7) Consider the following statements c. The excess of total government spending above
the national income
1. It is a mammal that can survive at an
altitude up to 5,000 metres. d. The Central bank‘s monetary policy strategy to
realise the objective of a stable rate of inflation
2. It has been found in the Himalayan
regions of India, Nepal, and Pakistan
and on the Qinghai-Tibetan Plateau of
10) Consider the following pairs
China.
1. Pong Dam wildlife sanctuary –
3. They also hibernate for more than six
Himachal Pradesh
months during the wintertime.
2. Sanjay Gandhi National Park –
4. It has been listed under the least
Maharashtra
concern category of IUCN Red list.
Which of the pair(s) given above is/are
Identify the species that correctly matches with
correctly matched?
the above description
a. 1 only
a. Reindeer
b. 2 only
b. Himalayan Marmot
c. Both 1 and 2
c. Hangul
d. Neither 1 nor 2
d. Snow Leopard

8) Consider the following statements with Answers


respect to Bio-toilets used in Indian railways 1. c
1. Inoculum is the bacteria used for bio-  The Rajasthan Panchayati Raj (amendment)
toilets to decompose waste. Bill, 2015, made Class X mandatory for
2. The bacteria used in bio-toilets are contesting municipal elections and for
currently being generated by the Indian contesting zila parishad or panchayat samiti
Railways. elections.

3. The bacteria used in bio-toilets  To contest the sarpanch elections, an aspirant


have very short expiry period, Once from the general category must have passed
opened, it has to be utilised within two Class VIII and a SC/ST aspirant must have
days. passed Class V.
Which of the statement(s) given above is/are  It has also made a functional toilet mandatory
correct? in the house of a contestant.
a. 1 and 3 only  By this, Rajasthan became the first State in
the country to fix a minimum educational
b. 2 and 3 only
qualification for contesting elections to the
c. 1, 2 and 3 Panchayati Raj Institutions.
d. None  The Rajasthan government
recently abolished the condition of a
minimum educational qualification to contest
9) The term ―Inflation Targeting‖ in local body elections.
economics refers to
2. a

www.shankariasacademy.com | www.iasparliament.com
 As part of the enhanced outreach programme recommendations of two different committees
of Indian Space Research Organisation led by Madhav Gadgil and K.
(ISRO), a new platform named ―Samwad with Kasturirangan.
Students‖ (SwS) was launched in Bengaluru
 ―Insensitivity towards ecology of Western
recently.
Ghats was making the six States of Western
 Through the SwS initiative, ISRO aims Ghats vulnerable to floods and landslides,‖ it
to constantly engage youngsters across noted.
India to capture their scientific temperament.
6. c
3. c
 India and Pakistan has recently exchanged
 The Regional Comprehensive Economic the list of nuclear installations and facilities
Partnership (RCEP), which is a proposed Free covered under the Agreement on the
Trade Agreement between ten ASEAN member Prohibition of Attack against Nuclear
states and their six Free Trade Agreement installations between them.
(FTA) partners namely India, Australia, China,
 Under the 1988 agreement, the two countries
Japan, New Zealand and Republic of Korea, is
inform each other of nuclear installations and
expected to provide market access for
facilities on the first of January every year.
India‘s goods and services
exports and encourage greater 7. b
investments and technology into India.
It would also facilitate India‘s MSMEs to  Researchers have sequenced the first complete
effectively integrate into the regional genome of the Himalayan marmot, which may
value and supply chains. help better explain how the mammals survive
at altitudes of up to 5,000 metres.
 In 2018, NITI Aayog has warned that, China‘s
capacity overhangs in most sectors and if  Marmots are found in the Himalayan
RCEP signed, it may lead to a surge of imports regions of India, Nepal, and Pakistan
into India with very limited access for Indian and on the Qinghai-Tibetan Plateau of
exports to the Chinese market. China, where many of them face extreme
cold, little oxygen, and few other resources.
4. b
 As one of the highest-altitude-dwelling
 UNESCO was co-founded by the U.S. after mammals, the Himalayan marmot is
World War II to foster peace. chronically exposed to cold temperature,
hypoxia, and intense UV radiation.
 Its headquarters is located in the Place de
Fontenoy, Paris.  They also hibernate for more than six
months during the wintertime.
 Recently, the United States and Israel
officially quit the U.N.‘s educational, 8. c
scientific and cultural agency with the close of
2018.  The Railway Convention Committee (RCC), in
its latest report has in particular sought a
 The countries had announced their decisions foolproof mechanism for the effective use and
in 2017 of withdrawing from the agency, management of the bacteria meant to be used
accusing it of bias against Israel. in the bio-toilets.
5. b  The bacteria Inoculum required for use in the
bio-toilets to decompose human waste is
 Over 56,000 km of ecologically sensitive areas
currently being generated by the
in the Western Ghats could not be earmarked
Railways at its in-house plant in Moti Bagh,
as ‗no-go‘ zones due to State governments‘
Nagpur, and is also procured from DRDO,
‗insensitivity‘, a parliamentary panel has
Gwalior.
recently stated.
 The committee detected a major problem with
 The panel had examined issues regarding the
the bacteria inoculums – it's short expiry
categorisation of Western Ghats parts as
date.
Ecologically Sensitive Areas (ESAs) as per the

www.shankariasacademy.com | www.iasparliament.com
 Once it is opened, it has to be utilised within b. Increasing Gross NPAs
two days, and the Railways is apparently not
c. Round Tripping
able to do so, rendering it ineffective.
d. FPI exodus
 It has sought a foolproof mechanism for the
effective use and management of the bacteria
meant to be used in bio-toilets. 3) Consider the following statements about
9. d Shola forest-grassland ecosystem
 The announcement of an official target range 1. It is a Tropical Montane Forest.
for inflation is known as inflation targeting. 2. It is characterised by patches of forest of
 It is done by the Central Bank in an stunted evergreen shola trees in the hill
economy as a part of their monetary slopes and grasslands on valleys.
policy to realise the objective of a stable 3. They are native to Southern Western
rate of inflation. Ghats.
 India commenced inflation targeting 4. Nilgiri Tahr is endemic to the region.
―formally‖ in February 2015 when an
agreement between the GoI and the RBI was Which of the statement(s) given above is/are
signed related to it – The Agreement on correct?
Monetary Policy Framework. a. 1 only
10. c b. 1, 2 and 3 only
 An ambitious eco-tourism project is all set to c. 1, 3 and 4 only
take wing in pong dam wildlife sanctuary
in Himachal Pradesh. d. 1 and 4 only

4) Consider the following statements with


respect to World Anti-Doping Agency (WADA)

03-01-2019 1. It is an international independent


agency funded equally by the sport
1) Consider the following statements with movement and governments of the
respect to swap ratios world.
1. It is the exchange rate at which the 2. Its primary activities include
currencies are accepted between two monitoring of the World Anti-Doping
countries with Currency Swap Code (Code).
Aggrement.
3. Pursuant to the terms of the Brazzaville
2. The swap ratios are based on stock Declaration, the World Anti-Doping
prices. Agency (WADA) was established.
Which of the statement(s) given above is/are Which of the statement(s) given above is/are
correct? correct?
a. 1 only a. 2 only
b. 2 only b. 1 and 2 only
c. Both 1 & 2 c. 1 and 3 only
d. Neither 1 nor 2 d. 1, 2 and 3

2) An expert committe under SEBI Chairman U 5) ―Asia Reassurance Initiative Act


K Sinha is set up to examine the solutions for? (ARIA)‖ was sometimes seen in the news
a. Sustainability of MSMEs

www.shankariasacademy.com | www.iasparliament.com
recently was enacted by which of the following Which of the statement(s) given above is/are
countries? correct?
a. China a. 1 only
b. France b. 1 and 2 only
c. India c. 1 and 3 only
d. United States of America d. 1, 2 and 3

6) Consider the following statements with 9) ―Io‖ was sometimes seen in the news
respect to National Health Authority recently. Consider the following statements
with respect to Io
1. It will dissolve and replace the existing
society “National Health Agency”. 1. It is one of the moons of planet Saturn.
2. It aims to efficiently implement 2. It is the most volcanically active world
Pradhan Mantri Annadata Aay in the Solar System.
SanraksHan Abhiyan (PM-AASHA).
3. Its extreme volcanic activity is due to
3. It will be an attached office under the the tug-of-war between Jupiter‟s gravity
Ministry of Health & Family Welfare. and its moons Europa and Ganymede‟s
gravity.
Which of the statement(s) given above is/are
correct? Which of the statement(s) given above is/are
correct?
a. 3 only
a. 2 only
b. 1 and 2 only
b. 1 and 2 only
c. 1 and 3 only
c. 2 and 3 only
d. 1, 2 and 3
d. 1, 2 and 3

7) ―Global Stocktake Process‖ was sometimes


seen in the news recently, is associated with 10) ―Moynagiri rice‖ is an indigenous rice
which of the following? variety of which of the following states?
a. Paris Agreement a. Assam
b. Kigali Agreement b. Maharashtra
c. Brazilia declaration c. Telangana
d. United Nations Environment Programme d. Tamilnadu
(UNEP)

Answers
8) Consider the following statements with
respect to Cinereous vulture 1. b

1. It is one of the heaviest and largest  When a company pays for an acquisition by
raptors in the world. issuing its own shares to the shareholders of
the target company, this is known as a share
2. It is classified as least concern in the swap.
IUCN Red list.
 The number of shares to be issued in lieu of
3. It migrates from the mountainous their existing holdings in the target company is
regions of Europe and Asia to India called the swap ratio.
during summer.
 The swap ratios are based on stock prices.

www.shankariasacademy.com | www.iasparliament.com
 It is determined by valuing the target company  The Brazzaville declaration was signed to
after looking into metrics such as its revenues promote better management and conservation
and profits, as well as its market price. world‘s largest tropical peatlands Cuvette
Centrale region in Congo basin from
unregulated land use and prevent its drainage
2. a and degradation.
 The panel was setup to suggest long-term 5. d
solutions for the economic and financial
 On the final day of 2018, U.S. President signed
sustainability of the MSME sector
into law the Asia Reassurance Initiative
 The eight-member committee will also Act (ARIA).
examine the factors impacting credit flow to
 The act establishes a multifaceted U.S.
MSMEs.
strategy to increase U.S.
 The report will be submitted by the end of security, economic interests, and values
June 2019. in the Indo-Pacific region.
3. c  It allocates a budget of $1.5 billion over a five-
year period to enhance cooperation with
 Shola forest-grassland ecosystem is America‘s strategic regional allies in
characterised by patches of forest of stunted the region.
evergreen shola trees in the valleys and
grasslands on hill slopes. 6. c
4. b  The Union Cabinet has recently approved the
restructuring of existing National Health
 WADA‘s key activities include scientific Agency as "National Health Authority" for
research, education, development of anti- better implementation of Pradhan Mantri -
doping capacities, and monitoring of the Jan Arogya Yojana (PM-JAY).
World Anti-Doping Code (Code) – the
document harmonizing anti-doping policies in  With this approval, the existing society
all sports and all countries. "National Health Agency" has been
dissolved and will be replaced by
 The First World Conference on Doping in National Health Authority as
Sport held, in Lausanne, Switzerland, on an attached office to Ministry of Health
February 2-4, 1999, produced the Lausanne & Family Welfare.
Declaration on Doping in Sport.
7. a
 Pursuant to the terms of the Lausanne
Declaration, the World Anti-Doping Agency  Article 14 of the Paris Agreement requires
(WADA) was established on November 10, the Conference of the Parties serving as the
1999, in Lausanne to promote and coordinate meeting of the Parties to the Paris Agreement
the fight against doping in sport (CMA) to periodically take stock of the
internationally. implementation of the Paris Agreement and to
assess collective progress towards achieving
 The WADA has recently said that Russian the purpose of the Agreement and its long-
authorities had failed to provide access to term goals. This process is called the global
laboratory doping data by their year-end stocktake.
deadline and it will consider sanctions against
the Russian Anti-Doping Agency (RUSADA).  The global stocktake shall be conducted in a
comprehensive and facilitative manner,
 RUSADA was stripped of its accreditation in considering mitigation, adaptation and the
2015 after a WADA-commissioned report means of implementation and support, and in
found evidence of widespread state- the light of equity and the best available
sponsored doping in Russian athletics. science.
 But it was conditionally – and controversially  The CMA will undertake the first global
– reinstated in September. stocktake in 2023 and every five years

www.shankariasacademy.com | www.iasparliament.com
thereafter, unless otherwise decided by the Assam known for its exceptional taste and
CMA. medicinal properties — was found
abundantly, before it started disappearing.
8. a
 At one point, the northeast had about 30,000
 It is one of the heaviest and largest
varieties of tholua (indigenous) paddy. Now,
raptors in the world.
many have gone extinct.
 It is classified as ―Near Threatened‖ in
 Green Revolution and the focus on the mono
IUCN Red list of threatened species.
crop led contributed to the decline.
 It is listed in Appendix II in CITES listings.
 Its geographical range expands from European
countries (such as Portugal, Spain, Croatia,
04-01-2019
Serbia etc), Middle East countries ( Turkey, 1) Consider the following statements
Lebanon, Saudi Arabia etc), Central Asia
(Russia, Azerbaijan, Georgia, Kazakhstan, 1. The retirement age of a judge of a High
Tajikstan, Iran etc), Asia (India, Afghanistan, Court is 65.
Myanmar, Korea, Vietnam). 2. A Bill, which sought to increase the
retirement age of High Court judges
 This species has a moderately small population
needs to be ratified by at least 50% of
which appears to be suffering an ongoing
the states.
decline in its Asiatic strongholds, despite the
fact that in parts of Europe numbers are now Which of the statement(s) given above is/are
increasing. correct?
 It generally occupies forest, shrubland and a. 1 only
grassland.
b. 2 only
 It is usually during the winter, it migrates c. Both 1 & 2
from the mountainous regions of Europe and
Asia to warmer places, including India. d. Neither 1 nor 2
 The species was recently spotted in the state of
Jharkhand for the first time in the region. 2) Consider the following statements
9. c about Clause 6 of Assam Accord

 Jupiter's rocky moon Io is the most 1. It was inserted to protect, preserve and
volcanically active world in the solar promote the culture and heritage of the
system, with hundreds of volcanoes, some “Assamese people”.
erupting lava fountains dozens of miles (or 2. It provides complete constitutional,
kilometers) high. legislative and administrative
 Io‘s remarkable volcanic activity is the result of safeguards to protect their socio-
a tug-of-war between Jupiter's powerful political rights.
gravity and smaller but precisely timed pulls Which of the statement(s) given above is/are
from two neighboring moons that orbit farther correct?
from Jupiter – Europa and Ganymede.
a. 1 only
 NASA’s solar-powered Juno spacecraft has
recently beamed back new images of volcanic b. 2 only
plume on Jupiter‘s moon Io, captured c. Both 1 & 2
during the mission’s 17th flyby of the gas
giant. d. Neither 1 nor 2

10. a
 Till the 1990s, moynagiri rice — an 3) ―Climate Change Performance Index
indigenous rice variety of lower (CCPI)‖ was published by which of the
following organisations?

www.shankariasacademy.com | www.iasparliament.com
a. UN Environment Programme (UNEP) 7) Consider the following statements with
respect to Kadaknath Chicken
b. Conservation International
1. It was also known as Kali Masi Chicken,
c. UN framework convention on climate change
traditionally reared by Jhabua adivasi
(UNFCCC)
communities of Odisha.
d. German watch
2. Its feathers, flesh and even bones are
black in colour.
4) ―Forest Reference Level (FRL) Report‖ was 3. It was granted Geographical Indication
sometimes seen in the news recently, is (GI) tag in 2018.
associated with which of the following?
Which of the statement(s) given above is/are
a. UN forum on Forests correct?
b. Project Tiger a. 3 only
c. UN convention to combat desertification b. 1 and 3 only
d. REDD+ c. 2 and 3 only
d. 1, 2 and 3
5) Consider the following statements with
respect to Change-4 Mission
8) ―The Von Karman Crater‖ was sometimes
1. It is China‟s mission to explore the dark seen in the news recently, is located in which of
side of the Moon. the following?
2. Since the moon's revolution cycle is the a. Mount Etna
same as its rotation cycle, one half of the
b. The Moon
moon will never expose to sunlight,
resulted in the dark zone of Moon. c. Anak krakatoa
3. With this mission, China has now joined d. Ultima Thule
U.S, Russia, EU and India who have
made a soft landing on the moon.
9) Consider the following statements with
Which of the statement(s) given above is/are
respect to Western Disturbances
correct?
1. It originates over the Mediterranean
a. 1 only
Sea, Caspian Sea and Black Sea and
b. 1 and 2 only move eastwards across north India.
c. 1 and 3 only 2. They are brought into India by the
easterly jet stream.
d. 1, 2 and 3
Which of the statement(s) given above is/are
correct?
6) ―World Development Report‖ was released
a. 1 only
by which of the following organisations?
b. 2 only
a. International Monetary Fund
c. Both 1 and 2
b. World Bank
d. Neither 1 nor 2
c. World Economic Forum
d. Organisation for Economic Co-operation and
Development 10) ―Astana Declaration‖ is related to which of
the following?
a. Road safety

www.shankariasacademy.com | www.iasparliament.com
b. Reduction of HFCs by Germanwatch and Climate Action
Network Europe.
c. Trans-boundary air pollution in South Asia
4. d
d. Primary Health Care
 A national forest reference (emission) level
(FRL) is one of the elements to be developed
Answers by developing country Parties
implementing REDD+ activities.
1. b
 As a first step towards accessing the potential
 The retirement age of Supreme Court judges is fund flows under REDD+, countries are
65 and of High Court judges is 62. required to set a baseline level of greenhouse
 Those provisions of the Constitution which are emissions being emitted due to deforestation
related to the federal structure of the polity can or prevented from escaping into the
be amended by a special majority of the atmosphere by conserving and growing forests.
Parliament and also with the consent of half of This is referred to as the Forest Reference
the state legislatures by a simple majority. Level or FRL.

 The following provisions can be amended in  Recently, UNFCCC has raised doubts over
this way: India’s claim of increase in forest cover, citing
lack of transparency in India’s FRL report.
1. Election of the President and its manner.
5. a
2. Extent of the executive power of the Union and
the states.  Change–4 is a Chinese space probe landed on
the dark side of the Moon.
3. Supreme Court and high courts.
 Since the moon's revolution cycle is the same
4. Distribution of legislative powers between the as its rotation cycle, the same side always faces
Union and the states. Earth.
5. Any of the lists in the Seventh Schedule.  The other face, most of which cannot be
6. Representation of states in Parliament. seen from Earth, is called the far side or
―dark side‖ of the Moon, not because it is
7. Power of Parliament to amend the dark, but because most of it is uncharted.
Constitution and its procedure (Article 368
 This is the first-ever soft landing on the dark
2. a side of the Moon.
 Clause 6 reads: ―Constitutional, legislative and  With this mission, China has now joined
administrative safeguards, as may be the U.S. and the former USSR as the only
appropriate, shall be provided to protect, countries to have made a ―soft landing‖ on
preserve and promote the cultural, social, the moon.
linguistic identity and heritage of the Assamese
people.‖ . 6. b
 However Clause 6 of the Assam Accord has  The World Development Report (WDR) is
not been fully implemented even almost an annual report published since 1978 by the
35 years after the Accord was signed. International Bank for Reconstruction and
Development (IBRD) or World Bank.
 So the Cabinet recently approved the setting
up of a High Level Committee to suggest  The recently released World Development
constitutional, legislative and administrative Report (WDR) 2019: the Changing
safeguards as envisaged in Clause 6 of the Nature of Work, studies how the nature of
Assam Accord. work is changing as a result of advances in
technology today.
3. d
7. c
 The Climate Change Performance Index
(CCPI) is an annual publication

www.shankariasacademy.com | www.iasparliament.com
 Kadaknath or Kali Masi (flowl with black  This would be through comprehensive
flash) is a unique breed of chicken that preventive, promotive, curative, rehabilitative
is completely black in colour. services and palliative care.
 It is completely black with black legs, beak,
tongue, comb and wattles.
05-01-2019
 Apart from its meat, its bones and most
organs are also black in colour. 1) Consider the following statements with
respect to Attorney General for India (AG)
 The chicken, traditionally reared by Jhabua‘s
1. He/She must be qualified to be
adivasi communities of Madhya
appointed a judge of the Supreme Court.
Pradesh, was granted a Geographical
Indication (GI) tag by the government 2. He/She is debarred from private legal
registry last year. practice.
8. b 3. He/She has the right to speak and to
take part in proceedings in any
 The Chinese probe Change-4 which has a committee of the parliament, even
lander and a rover, touched down at a targeted though he/she may not be a member of
area near the Moon‘s South Pole in the Von that committee.
Karman Crater.
Which of the statement(s) given above is/are
9. a correct?
 According to IMD glossary, western a. 1 only
disturbance is a weather disturbances noticed
as cyclonic circulation/trough in the mid and b. 1 and 3 only
lower tropospheric levels or as a low pressure c. 1, 2 and 3
area on the surface, which occur in middle
latitude westerlies. d. None
 It originates over the Mediterranean Sea,
Caspian Sea and Black Sea and move 2) Consider the following statements
eastwards across north India.
1. She was probably the modern India‟s
 The western cyclonic disturbances which enter first woman teacher.
the Indian subcontinent from the west and the
northwest during the winter months are 2. She along with her husband founded the
brought into India by the westerly jet first girl's school in Pune.
stream. 3. She had set up “Balhatya Pratibandhak
 An increase in the prevailing night Griha” to prevent female infanticide and
temperature generally indicates an advance in killing of widows and pregnant rape
the arrival of these cyclones disturbances. victims.

10. d Identify the personality who correctly matches


with the above description
 In October 2018, at Astana, Kazakhstan, world
leaders declared their commitment to a. Mary Mcleod Bethune
‗Primary Health Care‘. b. Savitribai Phule
 This declaration reaffirms the historic 1978 c. Pandita Ramabai Sarasvati
Declaration of Alma-Ata.
d. Begum Rokeya
 Alma-Ata Declaration was the first declaration
which identified Primary Health Care as a key
to the attainment of the goal of Health for All. 3) The ―X-Calibur instrument‖ sometimes seen
in the news recently, is used for which of the
 The Astana Declaration aims to meet all
following purposes?
people‘s health needs across the life course.

www.shankariasacademy.com | www.iasparliament.com
a. To detect Cosmic X-rays c. Both 1 and 2
b. To study Atmospheric neutrinos d. Neither 1 nor 2
c. To diagnose Xerophthalmia disease
d. To measure internal diameter of artilleries 7) The term ―Petroglyphs‖ was sometimes seen
in the news recently. It refers to
a. Carvings made on rocks
b. Handicrafts made of pebble
4) Consider the following statements with
respect to Silica c. Sculptures made of rocks
1. It is an odourless and a non-irritant d. Bacteria lives under rocks in extreme weather
compound. condition
2. Exposure to large amounts of silica dust
can cause silicosis disease, which is
8) Consider the following statements with
incurable.
respect to Swachh Survekshan 2019
Which of the statement(s) given above is/are
1. It is the fourth edition of the annual
correct?
cleanliness survey of both urban and
a. 1 only rural India.
b. 2 only 2. It was launched by the Ministry of
Housing and Urban affairs.
c. Both 1 and 2
Which of the statement(s) given above is/are
d. Neither 1 nor 2
correct?
a. 1 only
5) ―Yutu 2‖ was sometimes seen in the news
b. 2 only
recently. It is a/an
c. Both 1 and 2
a. Lunar rover of China‘s Change-4 mission
d. Neither 1 nor 2
b. Russia‘s mission to safeguard its cities from
melting of permafrost
c. China‘s non-nuclear weapon, dubbed as the 9) The term ―Orphan drugs‖ were sometimes
Mother of All Bombs seen in the news recently. It refers to
d. Updated version of USA‘s brain mimicking a. Drugs that are used to treat rare genetic
Super computer disorders
b. Drugs, for which the patent expires and is
allowed to replicate its chemical formula
6) Consider the following statements with
respect to Drugs (Prices Control) Amendment c. Drugs that are manufactured for research
Order, 2019 purposes and abandoned after its testing
1. It exempted new drugs patented under d. Drugs for which the pre-determined date
the Indian Patent Act from the price expires and is no longer be used for medical
control order for five years. purposes
2. It was issued by the Ministry of Health
and Family welfare.
10) Identify the ports, which are located in the
Which of the statement(s) given above is/are western coast of India
correct?
a. Kandla Port, Haldia Port, Dighi Port
a. 1 only
b. Kandla Port, Dighi Port, Marmagoa Port
b. 2 only

www.shankariasacademy.com | www.iasparliament.com
c. Krishnapatnam Port, Dighi port, Nhava Sheva  Exposure to large amounts of free silica can go
port unnoticed because silica is odourless, non-
irritant and does not cause any immediate
d. Ennore port, Haldia port, Krishnapatnam Port
health effects.
5. a
Answers  China has named the lunar rover,
1. a successfully deployed recently to carry out a
string of experiments on the far side of the
 He/She has the right to speak and to take part moon, as ‗Yutu-2‘.
in the proceedings of both the Houses of
Parliament or their joint sitting and any 6. a
committee of the Parliament of which  The Drugs (Prices Control) Amendment Order,
he may be named a member, but without 2019, by the Ministry of Chemicals and
a right to vote. Fertilisers exempts ―a manufacturer
 The Attorney General is not a full-time counsel producing a new drug patented under the
for the Government. He/She does not fall in Indian Patent Act, 1970, for a period of five
the category of government servants. years from the date of commencement of its
commercial marketing by the manufacturer in
 Further, he/she is not debarred from the country.
private legal practice.
 The provisions of DPCO 2013 shall not apply
2. b to drugs for treating orphan diseases
 Recently, Maharashtra government has (i.e) Orphan drugs to treat orphan
announced that a state-of-the-art skill diseases as decided by the Ministry of
development centre for women will be built at Health and Family welfare are
Naigaon in Satara district, named after social exempted from price control.
reformer Savitribai Phule, marking her 186th 7. a
birth anniversary.
 Petroglyphs are images created by removing
3. a part of a rock surface by incising,
 US scientists have successfully launched a picking, carving, or abrading, as a form
telescope - The X-Calibur instrument from of rock art.
Antarctica which analyses X-rays arriving  Andhra Pradesh‘s second largest petroglyph
from distant neutron stars, black holes site, containing about 80 petroglyhs, has been
and other exotic celestial bodies. discovered at Mekala Benchi, a spot near
 The instrument is carried aloft on a helium Aspari town, north of the Kurnool-Ballari
balloon intended to reach an altitude of highway in Kurnool district.
130,000 feet.  These petroglyphs, or rock carvings,
4. c underscore Kurnool‘s importance as a major
site of Neolithic settlements in south India.
 Silicosis is a form of occupational lung
disease caused by inhalation 8. b
of crystalline silica dust.  Minsitry of Housing and Urban affairs had
 Silicosis is a progressive interstitial lung recently launched the Swachh Survekshan
disease, characterized by shortness of breath, 2019 covering 4,237 towns and citiesin the
cough, fever and bluish skin. country.

 It occurs most commonly as an occupational  This digital and paperless survey is the fourth
disease of people working in the quarrying, edition of the annual cleanliness survey
manufacturing and building construction of "Urban India".
industries.  It is being conducted by an independent third
party.

www.shankariasacademy.com | www.iasparliament.com
9. a a. 1 only
 According to the Drugs (Prices Control) b. 2 only
Amendment Order, 2019, any drug maker
c. Both 1 & 2
who has brought in an innovative patented
drug will be exempt from the price control d. Neither 1 nor 2
regulations for five years since the start of
commercial marketing of the product in India.
3) Consider the following statements about
 This includes ‗orphan drugs‘ used to treat Border Roads Organisation (BRO)
rare genetic disorders.
1. It is engaged in road construction along
 Experts have observed that not bringing inaccessible regions in the border areas.
orphan drugs into price control will
significantly impact patients, as the treatment 2. It is staffed by personnel from Indian
[involving such drugs] typically costs crores Army, Military Police etc.
and lack of price control will have a 3. It functions under the Ministry of
detrimental effect on affordability. Defence.
10. b Which of the above statement(s) is/are
correct?

07-01-2019 a. 1 & 2 only

1) Consider the following statements about b. 1 & 3 only


Kurile Islands c. 2 & 3 only
1. It is an island archipelago in the d. All of the above
Pacific Ocean, lying between Russia and
Japan.
2. It is of volcanic origin. 4) ―MP Bezbarauah committee‖ was
sometimes seen in the news recently. It was
3. It comes under the jurisdiction of constituted for which of the following
Japan. purposes?
Which of the statement(s) given above is/are a. Issues with Start-ups
correct?
b. Implementation of Assam Accord
a. 1 only
c. Review dynamic pricing policy
b. 1 & 2 only
d. Investigate Meghalaya mine crisis
c. 2 & 3 only
d. All of the above
5) Consider the following statements with
respect to Agasthyarkoodam peak
2) Consider the following statements regarding 1. It is the second highest peak south of
Dynamic Pricing Policy satpura range.
1. This means that the prices fuels are 2. It is located within the Shendurney
changed daily by the Oil Marketing wildlife sanctuary.
Companies (OMCs) based on the
movement of international crude oil Which of the statement(s) given above is/are
prices. correct?

2. It is beneficial to both OMCs and the a. 1 only


consumers. b. 2 only
Which of the above statement(s) is/are c. Both 1 and 2
correct?

www.shankariasacademy.com | www.iasparliament.com
d. Neither 1 nor 2 b. Ministry of Women and Child Development
c. National Center for Missing and Exploited
Children
6) Consider the following statements with
respect to Mercury d. None of the above
1. It is the only metal to be liquid at room
temperature.
10) Consider the following statements with
2. It mixes with the environment due to respect to Phuti masjid
both natural and anthropogenic activity.
1. It was built by Sarfaraz Khan, the nawab
Which of the statement(s) given above is/are of Murshidabad.
correct?
2. It is renowned for its construction
a. 1 only which was completed in 5 years.
b. 2 only Which of the statement(s) given above is/are
correct?
c. Both 1 and 2
a. 1 only
d. Neither 1 nor 2
b. 2 only
c. Both 1 and 2
7) ―Financial Stability Report (FSR)‖ was
published by which of the following d. Neither 1 nor 2
organisations?
a. World Bank
Answers
b. World Economic Forum
1. b
c. International Monetary Fund
 The Kuriles are an archipelago of some 56
d. None of the above islands spanning about 1,800 km from Japan‘s
Hokkaido to Russia‘s Kamchatka.

8) Consider the following statements with


respect to Cyclone Pabuk
1. It originated over the Andaman Sea and
its neighbourhood.
2. The name Pabuk was given by
Bangladesh.
Which of the statement(s) given above is/are
correct?
a. 1 only
b. 2 only
c. Both 1 and 2
d. Neither 1 nor 2

9) “Mapping and Review Exercise of Child


Care Institutions Report‖ was released
recently by which of the following? 

a. Bachpan Bachao Andolan  All of them are under Russian jurisdiction but
Japan claims the two large southernmost

www.shankariasacademy.com | www.iasparliament.com
islands, Etorofu and Kunashiri, and two  It is the only metal to be liquid at room
others, Shikotan and Habomai, as its ‗northern temperature.
territories‘. ]
7. d
 These islands were occupied by the Soviet
Union in August 1945, after which the entire  The Reserve Bank of India (RBI) is
Japanese population, numbering less than releasing the Financial Stability Report
20,000, was evicted. bi-annually.

 Despite the passage of over 70 years, this  The Global Financial Stability Report (GFSR)
dispute has defied solution and prevented the is a semi-annual report by the International
conclusion of a Russo-Japanese peace treaty. Monetary Fund (IMF) that assesses the
stability of global financial markets and
2. c emerging-market financing.
3. d  It is released twice per year, in April and
4. b October.
8. d
 The government has notified a High-Level
Committee for implementation of Clause six of  Pabuk originated over the Gulf of Thailand
the Assam Accord. and its neighbourhood.
 The nine-member committee, comprising of  Pabuk is a name given by Laos and means a
eminent Assamese persons from different 'big fresh water fish'.
fields, is headed by former bureaucrat MP
Bezbarauah.  The India Meteorological Department (IMD)
has issued an Orange alert for the Andaman
 The committee will examine the effectiveness Islands as cyclonic storm Pabuk is moving
of actions since 1985 to implement Clause 6 of north-northwestwards.
the Assam Accord.
9. b
5. d
 Ministry of Women and Child Development
 Agasthyarkoodam is the second highest recently released a report titled ‗Mapping and
peak in Kerala at 1,868 metres in Review Exercise of Child Care Institutions‘.
the Neyyar Wildlife Sanctuary,
Thiruvananthapuram. 10. a

 The peak is named after a mythical Hindu sage  It was built by Sarfaraz Khan, the nawab
Agasthya. of Murshidabad.

 Kerala's Kani tribe has claimed that they  Phuti masjid is quite large with four cupolas at
have been traditionally worshipping the idol of the corners.
Agasthya Muni atop the hill and it was  Only two of its five planned domes were
customary for women not to go near the idol. completed. As the builder died soon after
 Last year, the Kerala High Court had ruled construction began, the mosque was never
that no gender-based restrictions can be completed.
imposed on those who intend to trek  And so the name Phuti Masjid, or broken
Agasthyarkoodam, and lifted the unofficial mosque. It is also known rather morbidly as
ban on the entry of women to the peak. Fouti Masjid. ‗Fout‘ means death, and the
6. c name was apparently given after the builder‘s
death.
 Mercury is a heavy metal that is
predominant in the environment.
 It mixes with the environment due to both 08-01-2019
natural (e.g. volcanic activity) 1) Consider the following statements regarding
and anthropogenic (e.g. electrical Vote on Account
appliances such as mercury lamps) activity.

www.shankariasacademy.com | www.iasparliament.com
1. It only represents the expenditure side 2. Indian Constitution does not provide for
of the government‟s budget. reservation on the basis of economic
criteria.
2. No direct tax proposals can be made via
vote on account. Which of the statement(s) given above
is/are incorrect?
3. It is also called as an Interim Budget.
a. 1 only
Which of the statement(s) given above
is/are not correct? b. 2 only
a. 1 and 2 only c. Both 1 and 2
b. 2 only d. Neither 1 nor 2
c. 3 only
d. All of the above 5) ―INS Kohassa‖ is sometimes seen in the
news recently. It is a/an
a. Ballistic missile submarine
2) Consider the following statements about
Neutron Stars b. Amphibious transport dock
1. Neutron stars are typically objects of c. Indian Navy‘s air base
very small radius composed
d. None of the above
predominantly of neutrons.
2. Neutron stars are formed by same
process as that of a Black hole. 6) Consider the following statements
Which of the statement(s) given above 1. It is contiguous to the protected areas of
is/are correct? Nagarhole and Bandipur of Karnataka
on the north-east and Mudumalai of
a. 1 only
Tamilnadu on the south-east.
b. 2 only
2. It is an integral part of the Nilgiri
c. Both 1 and 2 Biosphere Reserve.
d. Neither 1 nor 2 3. It is the second largest wildlife
sanctuary in Kerala.
Identify the wildlife sanctuary that correctly
3) Which of the following organizations
matches with the above description
receives the ―Exhibitor of the Year Award‖ at
the 106th Indian Science Congress held a. Nilgris wildlife sanctuary
recently?
b. Wayanad wildlife sanctuary
a. Indian Space Research Organisation (ISRO)
c. Mukurthi wildlife sanctuary
b. Council of Scientific & Industrial Research
d. Thekkady Wildlife Sanctuary
(CSIR)
c. National Centre for Biological Sciences (NCBS)
7) Consider the following statements with
d. Defence Research and Development
respect to Blue bottle Jelly fish
Organisation (DRDO)
1. They are known as bluebottles for their
transparent bluish appearance.
4) Consider the following statements
2. Bluebottle stings are painful but
1. In the Shreya Singhal v. Union of typically not life-threatening.
India case, the Supreme Court had
Which of the statement(s) given above is/are
capped the reservation at 50%.
correct?

www.shankariasacademy.com | www.iasparliament.com
a. 1 only 10) Consider the following statements with
respect to Gabon
b. 2 only
1. It is a landlocked country, located in
c. Both 1 and 2
East Africa.
d. Neither 1 nor 2
2. It is bordered by Cameroon, Republic of
Congo and Equatorial Guinea.
Which of the statement(s) given above is/are
correct?
8) Consider the following statements with
respect to Raisina Dialogue a. 1 only
1. It is a multilateral conference b. 2 only
committed to address the most
c. Both 1 and 2
challenging issues faced by Asian
Countries. d. Neither 1 nor 2
2. The theme of the Dialogue this year is “A
World Reorder: New Geometries; Fluid
Partnerships; Uncertain Outcomes”. Answers
3. The conference will be hosted by the 1. c
Centre for Asian Strategic Studies in  Budget has to be approved by Parliament
collaboration with the Ministry of before the commencement of the new financial
External Affairs. year.
Which of the statement(s) given above  But the discussion and passing of Budget
is/are incorrect? generally goes beyond the current financial
a. 2 only year.
b. 1 and 3 only  So a special provision called ―Vote on Account‖
is used, where the government obtains the vote
c. 2 and 3 only of Lok Sabha to withdraw money from CFI to
d. 1, 2 and 3 keep the money flowing for the government‘s
day to day functions, until the Budget is
passed. (Article 116)
9) Consider the following statements with
 e.g Salary to government employees, loan
respect to a River
interest payments, subsidies, pension
1. It originates on Ranchi plateau in payments etc.
Jharkhand and joins the Son River.
 It is also used in the years where Lok Sabha
2. Mandal Dam is being built across that elections are due.
River.
 But here vote on account is also presented
3. It meanders through the northern part along with minor policy changes, as it would
of Betla National Park. be improper for the outgoing government to
impose major budgetary constraints on its
Identify the river that correctly matches with
successor.
the above description
a. R. Lytein  Such a measure that includes both short term
expenditure and income part is called an
b. R. Subarnarekha Interim Budget.
c. R. North Koel 2. c
d. R. Barakar 3. d
 The Defence Research and Development
Organisation (DRDO) pavilion received

www.shankariasacademy.com | www.iasparliament.com
‗Exhibitor of the Year Award‘ at transparent bluish appearance) and is a
106th Indian Science Congress held at Lovely related, but distinct species to those found
Professional University (LPU), Phagwara, around the Atlantic.
Punjab.
 They are not "true" jellyfish, but belong to a
4. a group of organisms known
as siphonophores.
 The Union Cabinet has recently approved a
Constitution Amendment Bill to provide 10%  Bluebottle stings are painful but typically
reservation to the economically backward not life-threatening and can be treated
sections in the general category. with ice or hot water.
 The quota will be over and above the existing  Unusually strong winds pushed the jellyfish
50% reservation to the Scheduled Castes, the colonies close to shore.
Scheduled Tribes and the Other Backward
Classes (OBC). 8. b

 A nine-judge Constitutional Bench of the  The fourth edition of the Raisina Dialogue will
Supreme Court had, in the Indira Sawhney be held in New Delhi.
case, capped reservation at 50%.  Ministry of External Affairs in
 The constitutional amendment bill would be partnership with Observer Research
required as the constitution does not Foundation (ORF) is organizing the
provide for reservation on the ground flagship annual geopolitical and
of economic conditions. geostrategic conference.

 The amendment bill will envisages amendment  The theme of the Dialogue this year is "A
to Articles 15 and 16 of the Constitution. World Reorder: New Geometries; Fluid
Partnerships; Uncertain Outcomes‖.
5. c
 The discussions will seek to address issues
 The Indian Navy‘s air base, INS arising from ongoing global transitions and
Kohassa, will be commissioned by Navy changes to the world order, triggered by
Chief Admiral Sunil Lanba on January 24, unique leaders, innovative partnerships and
2019. new technologies.
 This will be India‘s fourth air base and the 9. c
third naval air facility in the archipelago,
which are closer to Southeast Asia than to the  Recently, PM Modi has laid the foundation
Indian mainland, overlooking key sea lanes of stone for the revival of North Koel (Mandal
communication and strategic choke points. Dam) project.

6. b 10. b

 With the rise in mercury in the Nilgiri  Gabon is a country on the


Biosphere, the seasonal migration of wild west coast of Central Africa.
animals from wildlife sanctuaries in  Located on the equator, Gabon is bordered
Karnataka and Tamil Nadu to the Wayanad by Equatorial Guinea to the
Wildlife Sanctuary (WWS) has begun. northwest, Cameroon to the north,
7. c the Republic of the Congo on the east and
south, and the Gulf of Guinea to the west.
 Highly venomous jellyfish have stung more
than three thousand people on Australia's
northeastern shores in just a few days that 09-01-2019
has led to the closure of several beaches in
Australia. 1) ―Nandan Nilekani committee‖ is sometimes
seen in the news recently. It was constituted for
 The bluebottle is also referred to as the Indo- which of the following purposes?
Pacific Portuguese man o' war (also
known as bluebottles for their a. To assess digitisation of payments

www.shankariasacademy.com | www.iasparliament.com
b. To review issues with Direct Benefit Transfer 4) The ―Global Report on Trafficking in
(DBT) scheme persons 2018‖ is released recently by which of
the following organisations?
c. To examine issues with biometrics-based
Aadhaar Programme a. Anti-slavery International
d. None of the above b. UN Office of Drugs and Crime (UNODC)
c. International Organization for Migration
(IMO)
2) NASA‟s probe Transiting Exoplanet Survey
Satellite (TESS) has recently discovered a new d. None of the above
planet. Which of the following are not the
criteria to designate a celestial body as a full-
sized planet? 5) Consider the following statements with
respect to Leptospirosis
1. It has to be perfectly spherical.
1. It is a bacterial disease that affects both
2. It has sufficient mass to assume
humans and animals.
hydrostatic equilibrium.
2. Human to human transmission of
3. It has cleared the neighbourhood
Leptospirosis occurs very rarely.
around its orbit.
Which of the statement(s) given above is/are
4. It has to reside within a zone of other
correct?
similarly-sized objects.
a. 1 only
Select the correct answer using the code given
below b. 2 only
a. 1 and 3 only c. Both 1 and 2
b. 1 and 4 only d. Neither 1 nor 2
c. 1, 2, 3 and 4
d. None 6) ―Global Economic Prospects Report‖ is
released by which of the following
organisations?
3) Consider the following statements with
a. World Bank
respect to Debenture Redemption Reserve
(DRR) b. International Monetary Fund
1. It is a provision mandating both listed c. World Economic Forum
and unlisted companies to set aside 25
per cent of their profits for protection of d. None of the above
bond investors in case of a default.
2. SEBI has recently scrapped the 7) Consider the following statements with
requirement of Debenture Redemption respect to Tea tribes
Reserve (DRR).
1. They are notified scheduled tribe
Which of the statement(s) given above is/are community, inhabited the state of
correct? Assam.
a. 1 only 2. They are entitled with the same benefits
b. 2 only that are granted to Scheduled Tribes in
other states.
c. Both 1 and 2
Which of the statement(s) given above is/are
d. Neither 1 nor 2 correct?
a. 1 only
b. 2 only

www.shankariasacademy.com | www.iasparliament.com
c. Both 1 and 2
d. Neither 1 nor 2 Answers
1. a
8) "Irisin" hormone sometimes seen in the  The Reserve Bank of India (RBI) has recently
news recently is generated by announced formation of a high-level panel
a. Pancreas to assess digitisation of payments in the
country.
b. Muscle tissue
 The Committee on Deepening of Digital
c. Pituitary gland Payments will be chaired by Nandan
d. Hypothalamus Nilekani.
 The committee will review the existing status
of digitisation of payments in the country and
9) Consider the following statements with assess the current levels of digital payments in
respect to Nuclear Power Corporation of India financial inclusion.
Ltd (NPCIL)
2. b
1. It is a Public Sector Enterprise.
 A planet is a celestial body that
2. It is responsible for design,
construction, commissioning and 1. Is in orbit around the Sun.
operation of nuclear power reactors in
2. Has sufficient mass for its self-gravity to
India.
overcome rigid body forces so that it assumes a
3. It comes under the administrative hydrostatic equilibrium (nearly round) shape.
control of the Department of Atomic
3. Has cleared the neighbourhood around its
Energy (DAE).
orbit.
Which of the statement(s) given above is/are
 A body need not be perfectly spherical
correct?
to be called a planet.
a. 2 only
 For example, the rotation of a body can slightly
b. 1 and 2 only distort the shape so that it is not perfectly
spherical.
c. 2 and 3 only
d. 1, 2 and 3  Earth, for example, has a slightly greater
diameter measured at the equator than
measured at the poles.
10) Consider the following statements with  Pluto now falls into the dwarf planet category
respect to Jal Charcha on account of its size and the fact that it
1. It is a weekly magazine issued by the resides within a zone of other similarly-
Ministry of Water Resources, River sized objects known as the transneptunian
Development & Ganga Rejuvenation. region.

2. It is a first of its kind initiative by the  NASA’s probe Transiting Exoplanet Survey
ministry to bring best practices in water Satellite (TESS) discovers a new planet.
sector to the national stage.  The planet, named HD 21749b, orbits a bright,
Which of the statement(s) given above is/are nearby dwarf star about 53 light years away,
correct? in the constellation Reticulum, and appears to
have the longest orbital period of the three
a. 1 only planets so far identified by TESS.
b. 2 only 3. a
c. Both 1 and 2  The finance ministry has rejected
d. Neither 1 nor 2 the Securities and Exchange Board of

www.shankariasacademy.com | www.iasparliament.com
India‘s (SEBI‘s) proposal to do away with  The communities are Koch-Rajbongshis, 36
the requirement of the debenture redemption Tea Tribes, Tai Ahoms, Morans, Motoks and
reserve (DRR). Chutiyas.
 DRR is a provision mandating both listed and  These communities are at present listed
unlisted companies to set aside 25 per cent as other backward classes (OBCs).
of their profits for protection of bond
investors in case of a default.  As per the information made available by the
National Commission for Backward Classes
 Financial institutions such as banks and non- (NCBC), there is no single
banking financial companies are, however, community described as ―Tea Tribe‖ in
exempted from this requirement if funds are the Central list of OBCs for the State of Assam.
raised through a private placement.
 However, there are as many as 96 castes in the
4. b Central List of OBCs for the State of Assam
falling under the description ―Tea Garden
 The 2018 UN Office of Drugs and Crime
Labourers, Tea Garden Tribes, Ex-Tea
(UNODC) Global Report on Trafficking in
Garden labourers & Ex-Tea Garden
Persons is the fourth of its kind mandated by
Tribes‖.
the General Assembly through the 2010 United
Nations Global Plan of Action to Combat 8. b
Trafficking in Persons.
 Irisin is a hormone generated by muscle
 Trafficking victims from South Asian tissue that is carried around the body in the
countries including India are detected in bloodstream.
many parts of Europe, according to a new UN
report that said human trafficking has taken  Irisin generated during physical exercise can
“horrific dimensions” with children help counter the loss of memory caused by
accounting for a third of those being Alzheimer's as well as delay the onset of the
trafficked. most common type of dementia, according to a
study.
5. c
 Researchers at the Federal University of Rio de
 Leptospirosis is a bacterial disease that Janeiro (UFRJ), established a link between
affects both humans and animals. levels of Irisin and the development of the
Alzheimer's disease.
 Humans become infected through direct
contact with the urine of infected animals or 9. d
with a urine-contaminated environment.
10. d
 The bacteria enter the body through cuts or
abrasions on the skin, or through the mucous  Ministry of Water Resources, River
membranes of the mouth, nose and eyes. Development and Ganga Rejuvenation has
released the inaugural issue of the monthly
 Person-to-person transmission is rare. magazine Jal Charcha – in New Delhi
recently.
6. a
 Jal Charcha is the third magazine being
 Global Economic Prospects is a World Bank
published by the Ministry.
Group flagship report that examines global
economic developments and prospects, with a  The first is Namami Gange by National
special focus on emerging market and Mission for Clean Ganga and second
developing economies. is Jalansh by Central Water Commission.
 It is issued twice a year, in January and June.  The magazine is also an effort to bring best
practices in water sector to the national stage
7. d
and move ahead in the direction of creating
 Centre has recently approved the proposal to water consciousness in the minds of the people
notify Six Communities from Assam as of the country.
Scheduled Tribes.

www.shankariasacademy.com | www.iasparliament.com
10-01-2019 Which of the statement(s) given above is/are
correct?
1) Consider the following statements with
respect to ―Web- Wonder Women‖ Campaign a. 1 only
1. It aims to recognise successful women b. 2 only
entrepreneurs in India and motivate c. Both 1 and 2
young girls to embrace
entrepreneurship as a career option. d. Neither 1 nor 2
2. It was launched by the Ministry of
Electronics and Information Technology 4) Parivara and Talawara communities
(MeitY). sometimes seen in the news recently, inhabited
Which of the statement(s) given above is/are which of the following states?
correct? a. Madhya Pradesh
a. 1 only b. Gujarat
b. 2 only c. West Bengal
c. Both 1 and 2 d. Karnataka
d. Neither 1 nor 2

5) ―NATPOLREX-VII‖ is sometimes seen in the


2) Consider the following statements with news recently. It refers to
respect to Tokenisation services a. National Polar Residence index launched by
1. It is the process of providing unique each Arctic Council members
alternate code, known as tokens in place b. National Level Pollution Response Exercise
of the actual card details. conducted by the Indian Navy
2. The Reserve Bank of India (RBI) has c. National Pollution Review exercise conducted
recently permitted authorised networks by the Central Pollution Control Board
to offer card tokenisation services.
d. None of the above
Which of the statement(s) given above is/are
correct?
a. 1 only 6) Consider the following statements with
respect to Development of North East Circuit
b. 2 only Tourism project
c. Both 1 and 2 1. It will be implemented under Swadesh
d. Neither 1 nor 2 Darshan Scheme of Ministry of
Tourism.
2. This is the first Swadesh Darshan
3) Consider the following statements project of Union Tourism Ministry in
1. The first specification of Scheduled the state of Meghalaya.
Tribes in relation to a particular State or Which of the statement(s) given above is/are
Union territory is by a notified Order of correct?
the President, after consultation with
the State Government / UT concerned. a. 1 only
2. Any subsequent inclusion in or b. 2 only
exclusion from and other modifications c. Both 1 and 2
in the list of Scheduled Tribes can be
made only through an amending Act of d. Neither 1 nor 2
Parliament.

www.shankariasacademy.com | www.iasparliament.com
7) Ministry of Agriculture & Farmers Welfare c. Jammu and Kashmir
has recently released recombinant Enzyme-
d. Meghalaya
linked immune sorbent assay (ELISA) kits for
which of the following diseases?
a. Celiac disease Answers
b. Type 1 diabetes 1. d
c. Rheumatoid arthritis  The Ministry of Women and Child
d. Equine Infectious Anaemia Development, Government of India, has
launched an online campaign, „#www: Web-
WonderWomen‟.
8) Consider the following statements with  The Campaign aims to discover and
respect to Jaitapur Nuclear Power plant celebrate the exceptional achievements of
1. It was planned to be established at women, who have been driving positive
Madhya Pradesh. agenda of social change via social media.
2. It will be established with the 2. c
cooperation of Russia.  The Reserve Bank of India (RBI) has
3. It consists of 6 European Pressurised permitted authorised card payment networks
Reactors (EPR) of 1650 MW each. to offer card tokenisation services to any third
party requesting it.
Which of the statement(s) given above is/are
correct?  Tokenisation acts as an additional layer of
security as it masks sensitive card
a. 3 only
data such as the 16-digit account number,
b. 1 and 3 only expiration date and security code.
c. 2 and 3 only  It creates a set of numbers that is used to
substitute your card information.
d. 1, 2 and 3
 This ensures that your card information is not
disclosed to the website or merchant you are
9) Consider the following statements with transacting with.
respect to Glanders disease
3. c
1. It is caused by bacterium and has
4. d
zoonotic potential.
2. It mainly affects equines including  On the basis of recommendation of State
horses, donkeys and mules. of Karnataka, it is proposed to amend
Constitution (Scheduled Tribes) Order, 1950
Which of the statement(s) given above is/are relating to Karnataka to grant Scheduled Tribe
correct? status to and to include the communities,
a. 1 only namely ―Parivara and Talawara as
Synonyms of ‗Nayaka‘ and ―Siddi‖
b. 2 only community of Dharwad and Belagavi districts
along with existing ‗Siddi‘ community of Uttar
c. Both 1 and 2
Kannada district in the list of Scheduled Tribes
d. Neither 1 nor 2 of Karnataka.
 This will fulfill the long term demand of
‗Parivara‘ and ‗Talawara‘ communities for
10) ―Umiam Lake‖ is located in which of the
granting Scheduled Tribes status in the State
following states?
of Karnataka.
a. Maharashtra
5. d
b. Kerala

www.shankariasacademy.com | www.iasparliament.com
 The seventh edition of  Glanders is a fatal infectious and notifiable
the National Level Pollution Response disease of equines including horses,
Exercise (NATPOLREX-VII) conducted by donkeys and mules.
the Indian Coast Guard held in the Arabian
Sea near Mumbai.  The disease is caused by a bacterium known
as Burkholderia mallei and has zoonotic
6. c potential.
 The Chief Minister of Meghalaya recently  The organism is also considered as potential
inaugurated the project ―Development of bio-weapon and categorized under ‗Tier 1
North East Circuit: Umiam (Lake View) Select Agent‘.
- U Lum Sohpetbneng- Mawdiangdiang -
Orchid Lake Resort‖ implemented under  After continuous research efforts of more than
Swadesh Darshan Scheme of Ministry of eight years, National Research Centre on
Tourism, Government of India. Equines (NRCE) has been able to develop a
recombinant Hcp1 antigen ELISA as an
 This is the first Swadesh Darshan alternate to complement-fixation test (CFT).
project of Union Tourism Ministry in
the state of Meghalaya. 10. d

7. d  The Chief Minister of Meghalaya recently


inaugurated the project ―Development of
 Union Ministry for Agriculture & Farmers North East Circuit: Umiam (Lake View)
Welfare has recently released recombinant - U Lum Sohpetbneng- Mawdiangdiang -
Enzyme-linked immune sorbent assay Orchid Lake Resort‖ implemented under
(ELISA) kits: one for Glanders and other Swadesh Darshan Scheme of Ministry of
for Equine Infectious Anaemia. Tourism, Government of India.
 Both these diseases are notifiable diseases in  This is the first Swadesh Darshan
India and require special diagnosis for control project of Union Tourism Ministry in
and eradication in the country. the state of Meghalaya.
 Equine infectious anaemia (EIA) is a chronic,
debilitating and persistent infectious disease of
equines caused by a retrovirus. 11-01-2019
 It is an OIE notifiable disease. NRCE has 1) Consider the following statements with
also developed recombinant p26 respect to Tax Inspectors without Borders
protein-based ELISA as an alternative to (TIWB) Programme
Coggin‟s test. 1. It facilitates the transfer of tax audit
knowledge and skills to developing
 This technology will provide sustainable and
homogeneous source of antigen and country tax administrations.
harmonized protocol to ensure regular 2. It is jointly launched by UNDP and
surveillance of EIA. OECD.
8. a 3. India is not a part of this programme.
 The JNPP project is proposed to be setup in Which of the statement(s) given above is/are
Ratnagiri district of Maharashtra. correct?
 It will be established with the cooperation a. 3 only
of France.
b. 1 and 2 only
9. c
c. 1 and 3 only
 Union Ministry for Agriculture & Farmers d. 1, 2 and 3
Welfare has recently released recombinant
Enzyme-linked immune sorbent assay
(ELISA) kits: one for Glanders and other
for Equine Infectious Anaemia.

www.shankariasacademy.com | www.iasparliament.com
2) ―Umesh Sinha Committee‖ sometimes seen 5) Consider the following statements
in the news recently was constituted for which
1. Tons and Giri rivers are the tributaries
of the following purposes?
of River Yamuna.
a. To study the impact of e-commerce on retail
2. Renukaji Dam Project is proposed to be
vendors
constructed on the river Giri in
b. To suggest changes to Representation of Himachal Pradesh.
Peoples Act, 1951
Which of the statement(s) given above
c. To review the working of University Grants is/are incorrect?
Commission
a. 1 only
d. None of the above
b. 2 only
c. Both 1 and 2
3) Consider the following statements with
d. Neither 1 nor 2
respect to National Clean Air Programme
(NCAP)
1. It will be a mid-term, 10 year action plan 6) Consider the following statements with
with 2019 as the first year. respect to Kharai Camels
2. It aims to reduce Particulate Matter 1. They are a unique breed of camel found
(PM) pollution by 20-30% across India, only in Kutch.
atleast by 2024.
2. They depend on mangroves for their
3. 2018 is considered as the base year for food.
this programme.
3. Industrialisation stands to be the
Which of the statement(s) given above is/are biggest threat for its existence.
correct?
Which of the statement(s) given above is/are
a. 2 only correct?
b. 2 and 3 only a. 1 only
c. 1, 2 and 3 b. 3 only
d. None c. 1 and 3 only
d. 1, 2 and 3
4) Consider the following statements
1. An amendment to a fundamental right 7) Consider the following statements with
coming under Part III does not require respect to Container Corporation of India
ratification by the legislatures of one (CONCOR)
half of the States.
1. It is a Public Sector Undertaking (PSU)
2. Principle of equality is one of the company, incorporated under the
foremost basic features of the Companies Act.
Constitution.
2. It comes under the Ministry of railways.
Which of the statement(s) given above is/are
correct? Which of the statement(s) given above is/are
correct?
a. 1 only
a. 1 only
b. 2 only
b. 2 only
c. Both 1 and 2
c. Both 1 and 2
d. Neither 1 nor 2
d. Neither 1 nor 2

www.shankariasacademy.com | www.iasparliament.com
Answers
8) Consider the following pairs 1. b
1. Hubble Telescope – U.S.A  The TIWB initiative intended to support
2. Kepler Telescope – U.S.A developing countries to strengthen national tax
administrations through building audit
3. CHIME Telescope – Canada capacity and to share this knowledge with
Which of the pair(s) given above is/are space other countries.
telescopes?  It is jointly launched by UNDP and OECD.
a. 2 only  The Union Cabinet has recently approved the
b. 1 and 2 only Signing of Terms of Reference (TOR)
governing the engagement of the designated
c. 1 and 3 only Indian Expert to provide tax assistance to
d. 1, 2 and 3 Swaziland (now known as Eswatini) under
the Tax Inspectors without Borders
Programme between India and Eswatini.
9) Consider the following statements with  India being a global leader in this respect has
respect to Swaziland a very important role to play in South-South
1. It is a landlocked country fully bordered Cooperation in tax matters.
by South Africa. 2. b
2. It was formerly called as Eswatini.  A Committee was constituted under the
Which of the statement(s) given above is/are chairmanship of Sr. Deputy Election
correct? Commissioner Sh. Umesh Sinha.
a. 1 only  It was constituted to review and suggest
modifications and changes in the
b. 2 only
provisions of the Section 126 and other
c. Both 1 and 2 sections of the Representation of the
People Act 1951, provisions of Model Code of
d. Neither 1 nor 2 Conduct and any other ECI instruction in this
regard.
10) Consider the following statements with  It has submitted its report recently.
respect to Indus Food 2019
3. d
1. It is India‟s export-focussed Food &
 The Centre has launched the National Clean
Beverage products trade fair.
Air Programme (NCAP) to reduce particulate
2. It is the second edition of the event that matter (PM) pollution by 20-30% in at least
held annually. 102 cities by 2024.
3. This year‟s theme is „World Food  This is not a pan-India, but a city-
Supermarket‟. specific programme.
Which of the statement(s) given above is/are  The NCAP will be a mid-term, five-year
correct? action plan with 2019 as the first year.
a. 1 only  For achieving the NCAP targets, the cities will
b. 1 and 3 only have to calculate the reduction in pollution
keeping 2017‘s average annual PM levels as
c. 2 and 3 only the base year.
d. 1, 2 and 3 4. c
 The proviso to Article 368 makes it clear that
when a Constitution amendment of a

www.shankariasacademy.com | www.iasparliament.com
fundamental right is in question, the Bill 20. Powers of the High Courts‘ under Articles 226
concerned need not be sent to the States‘ and 227.
Legislative Assemblies for ratification.
5. d
 Only Constitution amendments which affect
 Three storage projects are proposed to be
the Centre-State relations or division of powers
constructed on the river Yamuna and
in a federal structure require subsequent
two of its tributaries - Tons and Giri in
ratification by the States‘ Legislatures before
the hilly regions of Uttarakhand and Himachal
the Presidential assent.
Pradesh of Upper Yamuna Basin.
 Fundamental Rights and Directive
 These include Lakhwar project on river
Principles of State Policy can be amended
Yamuna in Uttarakhand , Kishau on river Tons
by a special majority of parliament and does
in Uttarakhand and Himachal
not require consent of states.
Pradesh and Renukaji on river Giri in
 From the various judgements, the following Himachal Pradesh.
have emerged as basic features of the
 An agreement for Renukaji Dam
Constitution or elements / components /
Multipurpose Project will be signed among six
ingredients of the ‗basic structure‘ of the
states - Uttar Pradesh, Haryana, Himachal
constitution:
Pradesh, Delhi, Rajasthan and Uttarakhand.
1. Supremacy of the Constitution
6. d
2. Sovereign, democratic and republican nature
of the Indian polity  The Kharai is a unique breed of camel
found only in Kutch, which feeds on
3. Secular character of the Constitution mangroves.
4. Separation of powers between the legislature,  Salt-making industries operating in the area
the executive and the judiciary are threatening its food supply and existence.
5. Federal character of the Constitution 7. c
6. Unity and integrity of the nation  CONCOR is a Navratna Public Sector
7. Welfare state (socio-economic justice) Undertaking (PSU) under the Indian
Ministry of Railways incorporated under
8. Judicial review the Companies Act.
9. Freedom and dignity of the individual  The maiden coastal voyage of SSL Mumbai, a
10. Parliamentary system vessel of Container Corporation of India
(Concor) was recently flagged off from Kandla
11. Rule of law Port to Tuticorin via Mangalore and Cochin.
12. Harmony and balance between Fundamental  This is a unique example of multi-modal
Rights and Directive Principles logistics solution through integrated rail
13. Principle of equality and coastal movement.

14. Free and fair elections  This modal shift in the logistics chain was an
important beginning that would help to bring
15. Independence of Judiciary down logistics costs and make Indian goods
16. Limited power of Parliament to amend the more competitive in the global markets in the
Constitution very near future.

17. Effective access to justice 8. b


18. Principles (or essence) underlying  Scientists have recently detected the second
fundamental rights. repeating fast radio burst (FRB) ever
recorded, a discovery that may help
19. Powers of the Supreme Court under Articles determine the origin of these mysterious
32, 136, 141 and 142 signals which have been linked with advanced
alien technology in the past.

www.shankariasacademy.com | www.iasparliament.com
 The discovery of this extragalactic signal was 2) Consider the following statements with
made through the Canadian Hydrogen respect to Women of India Organic Festival
Intensity Mapping Experiment (CHIME), an
1. It aims to celebrate and promote women
interferometric radio telescope in
farmers and entrepreneurs in the
the Dominion Radio Astrophysical
organic sector from the remotest parts
Observatory in British
of India.
Columbia, Canada. (It is not a space
telescope) 2. It has been held annually in New Delhi
since 2015.
9. d
3. It is being organized by the Ministry of
 Eswatini erstwhile known as Women and Child Development.
Swaziland is bordered by Mozambique to
its northeast and South Africa to its north, Which of the statement(s) given above is/are
west and south. correct?
 The Union Cabinet has recently approved the a. 1 only
Signing of Terms of Reference (TOR) b. 1 and 2 only
governing the engagement of the designated
Indian Expert to provide tax assistance to c. 2 and 3 only
Swaziland (now known as Eswatini) under d. 1, 2 and 3
the Tax Inspectors without Borders
Programme between India and Eswatini.
10. d 3) Sinho Commission report sometimes seen in
the news recently is associated with which of
 INDUS FOOD-II with the theme of ‗World the following?
Food Supermarket‘ will be held on 14th and
15th January, 2019, at India Expo Mart, a. Cabotage law in India
Greater Noida. b. Economically Backward Class welfare
 Organized annually, Indusfood is India‘s c. De-nationalization of banks in India
official, export-focussed F&B Trade fair
showcasing the country‘s best line-up of food & d. Impact of Gig economy in India
agri products to global buyers.

4) Consider the following statements


12-01-2019 1. Harappan Civilisation was a Bronze
1) Consider the following statements with Age civilisation mainly in the
respect to Partnership Summit 2019 northwestern regions of the Indian
subcontinent.
1. It is a global platform for dialogue,
debate, deliberation and engagement 2. Rakhigarhi is one of the most
among Indian and global leaders on prominent sites of the Harappan
terrorism. civilization located in Haryana, India.

2. It is being organized by the Ministry of Which of the statement(s) given above is/are
Home affairs. correct?

Which of the statement(s) given above is/are a. 1 only


correct? b. 2 only
a. 1 only c. Both 1 and 2
b. 2 only d. Neither 1 nor 2
c. Both 1 and 2
d. Neither 1 nor 2 5) Arrange the following hills from East to
West direction

www.shankariasacademy.com | www.iasparliament.com
1. Khasi Hills d. Paavakoothu
2. Jaintia Hills
3. Garo Hills 9) The term ―Adjournment Sine Die‖ in refers
to
Select the correct answer using the code given
below a. Terminating a sitting of Parliament for an
indefinite period
a. 1, 2, 3
b. Dissolution of the house, thereby ending the
b. 2, 1, 3
very life of the House
c. 2, 3, 1
c. Suspending the work in a sitting of Parliament
d. 3, 2, 1 for a specified time
d. The period spanning between the prorogation
of a House and its reassembly in a new session
6) H S Bedi Report sometimes seen in the news
recently, is associated with which of the
following?
10) The temperature of human body is
a. Anti-sikh riots regulated by which of the following?
b. Doubling farmers income by 2022 a. Epinephrine
c. Gujarat fake encounters b. Hypothalamus
d. None of the above c. Thymus
d. Norepinephrine
7) Rethi-Moraghat elephant corridor links
which of the following?
Answers
a. Rajaji National park with Betla Tiger Reserve
1. d
b. Silent Valley National park with Periyar Tiger
Reserve  The Partnership Summit is a global platform
for dialogue, debate, deliberation and
c. Nokrek National park with Kaziranga Tiger engagement among Indian and global
Reserve leaders on economic policy and growth
d. Gorumura National Park with Buxa Tiger trends in India.
Reserve  It is being organized by the Department of
Industrial Policy and Promotion,
Ministry of Commerce & Industry,
8) Consider the following statements Government of India, State Government of
1. It is a ritual dance drama from Kerala. Maharashtra and Confederation of Indian
Industry.
2. It found a place in the UNESCO‟s
intangible heritage list in 2010.  The 25th edition of Partnership Summit 2019
will be held from 13 January, 2019.
3. It is based on the mythological tale of a
battle between the goddess Kali and the 2. d
demon Darika.  The Ministry of Women and Child
Identify the art form that correctly matches Development is organizing the three-day
with the above description 6th ‗Women of India Organic Festival‘ from
tomorrow at Leisure Valley, Sector-10,
a. Kathakali Chandigarh, to celebrate and promote women
b. Mohiniyaatam farmers and entrepreneurs in the
organic sector from the remotest parts of
c. Mudiyettu India.

www.shankariasacademy.com | www.iasparliament.com
 The Women of India Organic Festival has  In December 2018, while studying the Rethi-
been held annually in New Delhi since Moraghat elephant corridor in the
2015. region, experts from the Wildlife Trust of India
(WTI) and others have discovered that
3. b
elephants of north Bengal have been
 The committee headed by chairman Maj. Gen. consuming plastic.
(Retd.) S.R. Sinho was constituted to suggest 8. c
criteria for identification for
economically backward classes as well  Mudiyettu is a glorious ritual
as recommend welfare measures and quantum dance performed in Kaali Temples
of reservation in education and Government across Kerala, predominantly in Central
employment to the extent as appropriate. Kerala (Ernakulam and Kottayam), celebrating
the triumph of Goddess Kaali over the
 The Sinho Commission report of 2010, demon Darikan.
has been cited by the Centre as the basis for its
legislation to grant 10 per cent reservation to  Mudiyettu found a place in UNESCO's
the Economically Weaker Sections (EWS) Representative list of Intangible
mainly among upper castes. Cultural Heritage of Humanity in 2010.
4. c 9. a
 Near-complete skeletal remains of a young  Adjournment sine die means terminating a
male and female have recently been discovered sitting of Parliament for an indefinite
at an archaeological site in Rakhigarhi period.
village in Haryana.
 In other words, when the House is adjourned
 Rakhigarhi was one of the most prominent without naming a day for reassembly, it is
sites of the Harappan civilisation. called adjournment sine die.
5. b  The power of adjournment as well as
adjournment sine die lies with the presiding
 Under North East Circuit of Swadesh Darshan
officer of the House
Scheme, the Ministry of Tourism has recently
sanctioned the project ‗Development of  An adjournment suspends the work in a
West Khasi Hills (Nongkhlaw- Krem sitting for a specified time, which may be
Tirot - Khudoi & Kohmang Falls – Khri hours, days or weeks.
River- Mawthadraishan, Shillong),
Jaintia Hills (Krang Suri Falls-  The period spanning between the prorogation
Shyrmang- Iooksi), Garo Hills (Nokrek of a House and its reassembly in a new session
Reserve, Katta Beel, Siju Caves)‟ in is called ‗recess‘.
Meghalaya. 10. b
6. c  The body maintains a constant temperature of
 The Special Task Force (STF) headed by around 37 degrees C because enzymes, the
Justice H S Bedi (retired) was asked to catalysts vital to body chemistry, function best
monitor the investigation of 17 encounter at around this temperature.
cases in Gujarat between 2002 and 2006.  The temperature is regulated by
 The Supreme Court has recently directed that the hypothalamus, which lies within the
a copy of Justice Bedi’s report should be given brain.
to the petitioners.  If the body heats up through exercise or
7. d illness, the hypothalamus promotes sweating
because evaporation of sweat cools the body.
 The Rethi-Moraghat elephant corridor,
which links Gorumara National Park  If body temperature falls, the hypothalamus
with Buxa Tiger Reserve, was crucial to sets in motion the mechanism for shivering
elephant movement. since shivering produces heat.

www.shankariasacademy.com | www.iasparliament.com
14-01-2019 a. 1 only
1) Consider the following statements with b. 2 only
respect to India-Central Asia Dialogue that held c. Both 1 and 2
recently
d. Neither 1 nor 2
1. It is the first ever India-Central Asia
Dialogue.
2. It is being held in Samarkand, 4) ―Legacy persons‘ clause‖ sometimes seen in
Kazakhstan. the news recently, is associated with which of
the following?
3. Afghanistan is also participating in the
dialogue. a. Sabarimala Temple entry issue
Which of the statement(s) given above is/are b. National Register for Citizens in Assam
correct? c. H1B Visas for Indian's in U.S
a. 2 only d. None of the above
b. 1 and 2 only
c. 1 and 3 only 5) Consider the following statements with
d. 1, 2 and 3 respect to National Youth Parliament Festival
2019
1. It will be conducted at three levels -
2) Consider the following statements with District, State and National.
respect to National Clean Energy Fund (NCEF)
2. The theme of this festival is “Be the
1. It was created out of cess on carbon Voice of New India and Find solutions
emission to provide financial support to and contribute to policy”.
clean energy initiatives.
3. It was being launched by the Ministry of
2. Union minister for Finance will approve Sports and Youth Affairs.
the project eligible for financing under
NCEF. Which of the statement(s) given above is/are
correct?
Which of the statement(s) given above is/are
correct? a. 2 only
a. 1 only b. 1 and 2 only
b. 2 only c. 2 and 3 only
c. Both 1 and 2 d. 1, 2 and 3
d. Neither 1 nor 2
6) ―Trishna wild life sanctuary‖ is located in
which of the following states?
3) Consider the following statements with
respect to Industrial Security Annex (ISA) a. Andhra Pradesh
agreement b. Uttar Pradesh
1. It was proposed to be signed between c. Tripura
India and Russia.
d. West Bengal
2. It would facilitate exchange of
geospatial information between the
parties for both military and civilian 7) Consider the following statements with
use.
respect to National Board for Wildlife
Which of the statement(s) given above is/are
correct?

www.shankariasacademy.com | www.iasparliament.com
1. It is a statutory organization constituted 1. He was a social reformer and was
under the Wildlife Protection Act, 1972. against religious dogmas.
2. It is chaired by Prime Minister of India. 2. He founded the Ramakrishna Mission in
1897.
Which of the statement(s) given above is/are
correct? 3. National leader‟s day was celebrated to
commemorate his birth anniversary.
a. 1 only
Which of the statement(s) given above
b. 2 only
is/are incorrect?
c. Both 1 and 2
a. 3 only
d. Neither 1 nor 2
b. 1 and 2 only
c. 1 and 3 only
8) Consider the following statements with
d. 1, 2 and 3
respect to The Energy Resources Institute
(TERI)
1. It conducts research works in the fields Answers
of energy, environment and sustainable
development. 1. c

2. It works under the administrative  The first ever India-Central Asia Dialogue
control of Ministry of New and which is expected to focus on a plethora of
Renewable Energy. regional issues including enhancing
connectivity to war-ravaged Afghanistan is
Which of the statement(s) given above is/are being held in Samarkand, the capital of
correct? Uzbekistan.
a. 1 only  The Foreign Minister of Afghanistan is also
b. 2 only participating in the dialogue as a special
invitee as the session is dedicated to
c. Both 1 and 2 connectivity issues in the region.
d. Neither 1 nor 2 2. d
 National Clean Energy Fund (NCEF) was
9) Consider the following statements created out of cess on coal produced /
imported (―polluter pays‖ principle) at Rs.
1. The Constitution (One Hundred and 400 per tonne to provide financial
Third Amendment) Act, 2019 reserves support to clean energy initiatives and
10 per cent quota for economically an Inter Ministerial Group chaired by
weaker sections. the Finance Secretary was constituted to
2. It grants reservation in both higher approve the project/schemes eligible for
education and government jobs. financing under NCEF.
Which of the statement(s) given above is/are  The 42nd standing committee on energy in its
correct? recent report on stressed gas-based power
plants has pulled up the government for
a. 1 only diverting coal cess to compensate States for
b. 2 only revenue loss post-GST, and recommended
financial support to the stressed gas-based
c. Both 1 and 2 power projects in the country from National
d. Neither 1 nor 2 Clean Energy Fund (NCEF).
3. d

10) Consider the following statements with  It was planned to be signed between India
respect to Swami Vivekananda and the U.S.

www.shankariasacademy.com | www.iasparliament.com
 It allows sharing of classified information from  As the gas bearing zones are in the wildlife
the U.S. government and American companies sanctuary, permission from the National
with the Indian private sector, which is so far Wildlife Board is necessary.
limited to the Indian government and the
8. a
defence public sector undertakings.
 TERI is a Non-Profit Research
 BECA refers to Basic Exchange and
institute located in Delhi, established
Cooperation Agreement for Geo-spatial
in 1974 and formally known as Tata Energy
Cooperation.
and Resource Institute.
 This agreement would facilitate exchange of
 Its activities have scope in climate change,
geospatial information between India and
energy efficiency, renewable energy,
United States for both military and civilian
biotechnology and social transformation.
use.
9. c
4. b
 Recently, Indian President Ram gave his
 The National Register of Citizens (NRC) is the
assent to the The Constitution (124
list of Indian citizens of Assam.
Amendment) Bill, 2019.
 It was prepared in 1951, following the census of
 The Constitution (103rd)
1951.
Amendment Act, 2019 provides for 10%
 For a person‘s name to be included in the reservation in education and government
updated NRC list of 2018, he/ she will have to jobs for economically weak from upper castes.
furnish:
10. a
1. Existence of name in the legacy
 National Youth Day (on every year,
data: The legacy data is the collective list of
January 12) was celebrated to commemorate
the NRC data of 1951 and the electoral rolls up
the birth anniversary of Swami Vivekananda.
to midnight of 24 March 197
2. Proving linkage with the person whose
name appears in the legacy data. 16-01-2019
 A legacy person is someone who figures in a set 1) Consider the following statements with
of pre-1971 documents such as the 1951 NRC respect to Global Aviation Summit 2019
and voters‘ lists up to 1971, who an applicant
can trace his or her lineage to. 1. It is being held in New Delhi, India.

5. d 2. It is being organized jointly by the


Ministry of Civil Aviation and the
6. c International Civil Aviation
Organization (ICAO).
 Trishna Wildlife Sanctuary is
located in Tripura. 3. The theme for the event is „Flying for all:
especially the next 6 Billion‟.
 Recently, National Wildlife Board has cleared
the ONGC Trishna gas project in Trishna Which of the statement(s) given above is/are
Wildlife Sanctuary. correct?
 As the gas bearing zones are in the wildlife a. 3 only
sanctuary, permission from the National
b. 1 and 2 only
Wildlife Board is necessary.
c. 1 and 3 only
7. c
d. 1, 2 and 3
 Recently, National Wildlife Board has cleared
the ONGC Trishna gas project in Trishna
Wildlife Sanctuary.
2) Consider the following statements

www.shankariasacademy.com | www.iasparliament.com
Assertion (A): Blood Moon phenomenon 2. They are part of the Electoral Literacy
happens during a total lunar eclipse, when the Club (ELC) programme of Election
sun, Earth and moon are perfectly lined up. Commission of India (ECI).
Reason (R): The little light that manages to Which of the statement(s) given above is/are
pass from the edges of the Earth‟s atmosphere correct?
lit up the Moon‟s surface and makes it look red.
a. 1 only
In the context of the statements given above,
b. 2 only
which of the following is true?
c. Both 1 and 2
a. A and R both are true, and R is the correct
explanation for A d. Neither 1 nor 2
b. A and R both are true, and R is the not the
correct explanation for A
6) DAMINI app which was recently launched by
c. A is correct, R is incorrect Indian Institute of Tropical Meteorology
(IITM), Pune to give alerts on
d. A and R both are incorrect
a. Lightning
b. Cyclones
3) ―Annual Status of Education (ASER)
report‖ is prepared by which of the following? c. Heat waves
a. Ministry of Human Resource and d. None of the above
Development
b. Ministry of Women and Child Development
7) Consider the following statements with
c. NITI Aayog respect to Womaniya on GeM initiative
d. None of the above 1. It enables women entrepreneurs and
women self-help groups to sell their
products directly to various
4) Consider the following statements with Government institutions through the
respect to Monkey fever Government eMarketplace
(GeM) platform.
1. It is a tick-borne viral hemorrhagic
fever endemic to Indian Subcontinent. 2. It was launched by the Ministry of
Finance.
2. It is caused by Rhino virus, a member of
the virus family Flaviviridae. Which of the statement(s) given above
is/are incorrect?
Which of the statement(s) given above is/are
correct? a. 1 only
a. 1 only b. 2 only
b. 2 only c. Both 1 and 2
c. Both 1 and 2 d. Neither 1 nor 2
d. Neither 1 nor 2
8) In which of the following states Roshni
Act was repealed recently?
5) Consider the following statements with
respect to Voter Awareness Forums (VAFs) a. Chhattisgarh
1. It is an online portal launched to b. Manipur
generate awareness around electoral
c. West Bengal
process to the common masses.
d. Jammu & Kashmir

www.shankariasacademy.com | www.iasparliament.com
 However, the little light that manages to pass
from the edges of the Earth‘s atmosphere lit up
9) Kanhoji Angre island sometimes seen in the
the Moon‘s surface and makes it look red.
news recently, is located along which of the
following state‟s coast?  It is due to the red appearance of the Moon
a. Goa during the total lunar eclipse that it is called
―Blood‖ Moon.
b. Odisha
 Super Blood Moon will appear in the night
c. Maharashtra sky around much of the world on January 20
d. Andhra Pradesh and 21.
3. d

10) River Darling and Lake Hume sometimes  The Annual Status of Education Report
seen in the news recently are located in which (ASER) is an initiative by NGO Pratham.
of the following countries?  Annual Status of Education (ASER) report
a. Italy 2018 was released recently.
b. Colombia 4. d
c. Australia  According to National Health
Portal, Kyasanur forest disease (KFD) is a
d. United States of America tick-borne viral haemorrhagic
fever endemic(constant presence of
disease) in Karnataka State, India.
Answers
 It is also referred as monkey fever by local
1. a people.
 Global Aviation Summit 2019, the first of its  The virus causing the disease: KFD virus
kind initative was inaugurated in Mumbai, (KFDV) is a member of the genus Flavivirus
India recently. and family Flaviviridae.
 The two-day summit, with a theme 'Flying  The rhinovirus is the most common viral
for all-especially the next 6 Billion', is infectious agent in humans and is the
being organized by Ministry of Civil predominant cause of the common cold.
Aviation in association with FICCI.
5. b
 The Summit is supported by International
Civil Aviation Organization (ICAO) and other  Voter Awareness Forums
agencies. (VAFs) are informal forums for generating
discussions and awareness around the
2. a electoral process, on the how, what and where
of registration & voting, through the medium
 Lunar eclipses occurs when Earth's shadow
blocks the sun's light, which otherwise reflects of real-time activities.
off the moon.  Through VAFs in Government Departments,
 There are three types — total, partial and Government and Non-Government
penumbral — with the most dramatic being a Organizations as well as in Corporates; the
total lunar eclipse, in which Earth's shadow Election Commission of India aims to spread
completely covers the moon. voter awareness and facilitate voter education.

 During this phenomenon, the Sun, Earth  VAF is part of the Electoral Literacy
and the Moon are perfectly lined up with Club (ELC) programme of ECI.
the Earth blocking the Sun‘s light from falling  The ELC programme envisages setting up of
directly on the Moon. Electoral Literacy Club in every educational
institution and Chunav Pathshala at every

www.shankariasacademy.com | www.iasparliament.com
booth to cover those outside the formal  Recently Mumbai Port Trust has announced
education system. that it will start a ferry service from the
domestic terminal to the Kanhoji Angre
6. a
Island.
 IITM Pune developed a Mobile App called
 Currently, there is no direct ferry service to
―DAMINI‖ to give alerts on impending
Kanhoji Angre Island as the water level is very
lightning activity over the area.
low during the low tide.
7. b
 The boat services may attract more tourists as
 Government e Marketplace [GeM] has it will reduce the time to reach the island.
launched ―Womaniya on GeM‖, an initiative
10. c
to enable women entrepreneurs and women
self-help groups [WSHGs] to sell handicrafts  Up to a million dead fish were found floating
and handloom, accessories, jute and coir recently in the Darling River in western
products, home décor and office New South Wales state of Australia.
furnishings, directly to various Government
ministries, departments and institutions.  Also, more rotting fish had since been found
in Lake Hume in the state‘s south.
 The initiative seeks to develop women
entrepreneurship on the margins of  The state government has recently announced
society to achieve gender-inclusive plans to mechanically pump oxygen into
economic growth. lakes and rivers after hundreds of
thousands of fish have died.
 Government e Marketplace is a 100 percent
government owned company setup under the  Experts blame heat wave conditions
aegis of Ministry of Commerce and across much of Australia, drought and
Industry for procurement of common use algal blooms for starving waterways of
goods and services by Government ministries, oxygen.
departments and CPSEs.
8. d
17-01-2019
 The State Administrative Council (SAC) 1) Consider the following statements with
headed by Jammu & respect to Gandhi Peace Prize
Kashmir Governor Satya Pal Malik recently
repealed the Jammu and Kashmir State 1. The award will be conferred annually
Lands (Vesting of Ownership to the and is open to people of all nationalities.
Occupants) Act, 2001, popularly known as
2. Gandhian peace prize for the year 2018
the Roshni Act.
was conferred on the Indian Space
 The Act envisaged the transfer of ownership Research Organisation (ISRO).
rights of state land to its occupants, subject to Which of the statement(s) given above is/are
the payment of a cost, as determined by the correct?
government.
a. 1 only
 But, it had failed to realize the desired
objectives and there were also reports of b. 2 only
misuse of some its provisions, leading to its c. Both 1 and 2
repeal.
d. Neither 1 nor 2
9. c
 Kanhoji Angre Island is located south of
Gateway of India and acts as an entry point to 2) Consider the following statements with
the Mumbai harbour. respect to Sangeet Natak Akademi (SNA)

 It further acting as prime aid for navigation of 1. It is a statutory body works under the
vessels arriving and departing from Mumbai. Ministry of Culture.

www.shankariasacademy.com | www.iasparliament.com
2. It is the nodal agency of the Ministry of 5) Consider the following statements with
Culture to coordinate the matters respect to IMBEX 2018-19
related to Intangible Cultural Heritage
1. It is a bilateral Army exercise between
and various UNESCO Conventions
India and Myanmar.
addressing Cultural Diversity.
2. It seeks to improve tactical and
Which of the statement(s) given above is/are
technical skills in joint counter
correct?
insurgency and counter terrorist
a. 1 only operations.
b. 2 only Which of the statement(s) given above is/are
correct?
c. Both 1 and 2
a. 1 only
d. Neither 1 nor 2
b. 2 only
c. Both 1 and 2
3) Consider the following statements with
respect to Congenital Central Hypoventilation d. Neither 1 nor 2
Syndrome (CCHS)
1. It is a disorder of the nervous system, in
6) “Macedonia naming dispute‖ sometimes
which the cue to breathe is lost when the
seen in the news recently, is a dispute between
patient goes to sleep.
which of the following two countries?
2. Vaccination at an earlier stage can
a. Slovenia and Macedonia
restrict its incidence in humans.
b. Albania and Republic of Macedonia
Which of the statement(s) given above is/are
correct? c. Macedonia and Republic of Macedonia
a. 1 only d. Greece and Republic of Macedonia
b. 2 only
c. Both 1 and 2 7) Consider the following statements with
respect to Index of Industrial Production (IIP)
d. Neither 1 nor 2
1. It is a composite indicator that
measures long term changes in the
4) Consider the following statements volume of production of a basket of
industrial products with respect to given
1. River Zanskar is a north flowing
base year.
tributary of the Indus River in Jammu
and Kashmir. 2. It is compiled and published by the
Central Statistics Office (CSO).
2. India‟s one of the toughest trails,
Chadar Trek was on the frozen zanskar Which of the statement(s) given above is/are
river. correct?
Which of the statement(s) given above is/are a. 1 only
correct?
b. 2 only
a. 1 only
c. Both 1 and 2
b. 2 only
d. Neither 1 nor 2
c. Both 1 and 2
d. Neither 1 nor 2
8) Consider the following statements with
respect to Exim Bank of India (Exim Bank)

www.shankariasacademy.com | www.iasparliament.com
1. It was established in 1982 under an Act  It is open to people of all nationalities. The
of Parliament. award was jointly given in 2000 to Nelson
Mandela and Grameen Bank of Bangladesh.
2. It is regulated by Reserve Bank of India
(RBI).  After a gap of four years, the Centre recently
Which of the statement(s) given above is/are announced the awardees for the prestigious
correct? Gandhi Peace Prize for 2015 to 2018.

a. 1 only  For the year 2018, it was conferred on WHO


Goodwill Ambassador Yohei Sasakawa for his
b. 2 only role in leprosy eradication.
c. Both 1 and 2 2. b
d. Neither 1 nor 2  The Sangeet Natak Akademi (SNA), the
National Academy of Music, Dance and Drama
is an autonomous body under
9) SANJHI –MUJH MEIN KALAKAR - an the Ministry of Culture, Govt. of India.
initiative to document and promote the
Intangible Cultural Heritage (ICH) was  It is the nodal agency of the Ministry of
launched by which of the following? Culture to coordinate the matters
related to Intangible Cultural
a. Anthropological Survey of India (AnSI) Heritage and various UNESCO Conventions
b. Centre for Cultural Resources and Training addressing Cultural Diversity and promotion
(CCRT) and dissemination of multifarious cultural
traditions and expressions of the country.
c. Indira Gandhi National Centre for Arts
 Recently, Sangeet Natak Akademi (SNA) has
d. None of the above launched the second phase of the Web
Campaign ‗SANJHI –MUJH MEIN
KALAKAR‘ an initiative to document and
10) Krushak Assistance for Livelihood and promote the Intangible Cultural Heritage
Income Augmentation (KALIA) Scheme was (ICH) and diverse cultural traditions of the
recently launched in which of the following country by direct public-participation.
states?
3. a
a. Uttar Pradesh
 CCHS (also known as Ondine‘s Curse) is
b. Odisha a disorder of the nervous system in
c. Telangana which the cue to breathe is lost when the
patient goes to sleep.
d. West Bengal
 This results in a lack of oxygen and a build-up
of carbon dioxide in the body, which can
Answers sometimes turn fatal.

1. a  According to NIH, Treatment typically


includes mechanical ventilation or use of
 The International Gandhi Peace Prize was a diaphragm pacemaker.
instituted by the government on the 125th
birth anniversary of Mahatma Gandhi. 4. c

 It is given to individuals and institutions for  The Chadar Trek is one of the toughest and
their contribution towards social, economic most challenging trails in the country.
and political transformation through non-  This arduous expedition was on the frozen
violence and other Gandhian methods, was last Zanskar river.
conferred upon the Indian Space Research
Organisation in 2014.  The Zanskar River is a north flowing
tributary of the Indus River in Jammu
and Kashmir.

www.shankariasacademy.com | www.iasparliament.com
 Air force Station Thoise has recently 7. b
conducted a trekking expedition in this
 IIP is a composite indicator that measures
daunting terrain of Ladakh region to
the short-term changes in the volume of
commemorate the momentous occasion of
production of a basket of industrial products
70th Republic Day.
during a given period with respect to that in a
5. a chosen base period.
 The second India-  It is compiled and published monthly by
Myanmar bilateral Army exercise 'IMBEX the Central Statistics Office (CSO) with a
2018-19' has recently commenced at time lag of six weeks from the reference
Chandimandir military station, Chandigarh, month.
Haryana.
 For the annual revised estimates, the CSO used
 The aim of this joint training event is to train to replace the IIP with the ASI (Annual Survey
the Myanmar delegation for of Industries), which comes out with a two-
participation in UN peacekeeping year lag.
operations.
8. c
 The Indian Army has a rich experience of
 Exim Bank is the principal export credit
serving in UN peacekeeping operations, being
agency for India.
the foremost troop contributing country
in the UN.  It was established in 1982 under an Act of
6. d Parliament as the apex financial institution
for financing, facilitating and promoting
 Macedonia naming dispute is a longstanding India's international trade.
dispute between Greece and the Republic
of Macedonia.  The Bank primarily lends for exports from
India including supporting overseas buyers
 Because, a region of Greece bordering and Indian suppliers for export of
the Macedonian republic is also called developmental and infrastructure projects,
Macedonia. equipment, goods and services from India.
 Recently, Parliamentarians of the Republic  It is regulated by RBI.
of Macedonia voted to change their
country‘s name to the ―Republic of North  The Union Cabinet has recently approved the
Macedonia‖. recapitalization of EXIM Bank.
 The infusion of capital into Exim Bank will
enable it to augment capital adequacy and
support Indian exports with enhanced ability.
9. d
 Recently, Sangeet Natak Akademi (SNA) has
launched the second phase of the Web
Campaign ‗SANJHI –MUJH MEIN
KALAKAR‘ an initiative to document and
promote the Intangible Cultural Heritage
(ICH) and diverse cultural traditions of the
country by direct public-participation.
 This is a unique talent search where the
participant will showcase their talent in the
fields of music, dance, drama, puppetry, folk
and tribal arts, culinary skills, painting,
sculpture etc.
10. b

www.shankariasacademy.com | www.iasparliament.com
 The government of Odisha has launched the 1. Under the scheme, the Government
Krushak Assistance for Livelihood and Income incentivize employers‟ based on their
Augmentation (KALIA) scheme, which would employment generation potential.
benefit about 92% of the farmers in the State.
2. It is being implemented by Ministry of
 It will accelerate agricultural prosperity and Labour and Employment through the
elimination of poverty. Employees‟ Provident Fund
Organization (EPFO).
Which of the statement(s) given above is/are
18-01-2019 correct?
1) Consider the following statements with a. 1 only
respect to SAKSHAM 2019
b. 2 only
1. It is an annual, month-long mass
awareness campaign on fuel c. Both 1 and 2
conservation. d. Neither 1 nor 2
2. It was launched by the Petroleum
Conservation Research Association
(PCRA). 4) ―Zakiullah Khan Committee‖ sometimes
seen in the news recently was constituted for
Which of the statement(s) given above is/are which of the following purposes?
correct?
a. To suggest guidelines for space technology in
a. 1 only Border management
b. 2 only b. To study the contemporary status of Muslims
c. Both 1 and 2 in India

d. Neither 1 nor 2 c. To review inmates‘ condition in jails and


suggest reform measures
d. None of the above
2) Consider the following statements with
respect to UNNATI programme
1. It is a capacity building programme on 5) Consider the following statements
developing heavy satellites to provide 1. The Group of 77 is the largest
internet connectivity to remote areas. intergovernmental organization of
2. It is an initiative by ISRO to developing countries in the United
commemorate the 50th anniversary of Nations.
the first United Nations conference on 2. Palestine recently assumed the
the exploration and peaceful uses of chairmanship of G77 grouping for the
outer space (UNISPACE-50). year 2019.
Which of the statement(s) given above is/are 3. Palestine is a non-member observer
correct? State at the United Nations.
a. 1 only Which of the statements given above are
b. 2 only correct?

c. Both 1 and 2 a. 1 and 2 only

d. Neither 1 nor 2 b. 1 and 3 only


c. 2 and 3 only

3) Consider the following statements with d. 1, 2 and 3


respect to Pradhan Mantri Rozgar Protsahan
Yojana (PMRPY)

www.shankariasacademy.com | www.iasparliament.com
6) Consider the following statements with will not be a member of the selection
respect to Celiac disease committee?
1. It is an autoimmune disease. 1. The President
2. It occurs because of ingestion of a 2. The Prime Minister
protein, called gluten.
3. The Speaker of the Rajya Sabha
3. It damages the liver and resulted in
4. The Leader of the Opposition in the
improper nutrient absorption.
Rajya Sabha
Which of the statements given above are
5. An eminent jurist
correct?
6. Attorney General of India
a. 2 only
Select the correct answer using the code given
b. 1 and 2 only
below
c. 1 and 3 only
a. 1 and 3 only
d. 1, 2 and 3
b. 3 and 4 only
c. 1, 3, 4 and 6 only
7) Which of the following states is the first to
d. All of the six
launch the Cyber Security portal in India?
a. Haryana
10) Consider the following statements with
b. Telangana
respect to Cassini mission
c. Karnataka
1. It was sent to study Saturn and its
d. Andhra Pradesh complex system of rings and moons in
unprecedented detail.
2. It is a joint endeavor of NASA, ESA (the
8) Consider the following statements with
European Space Agency) and the Italian
respect to Petroleum Conservation Research
space agency (ASI).
Association (PCRA)
Which of the statement(s) given above is/are
1. It assist in developing policies which
correct?
promote energy efficiency in various
sectors of economy within the overall a. 1 only
framework of the Energy Conservation
b. 2 only
Act, 2001.
c. Both 1 and 2
2. PCRA is a non-profit organization, set
up under the aegis of Ministry of d. Neither 1 nor 2
Petroleum and Natural Gas.
Which of the statement(s) given above is/are
correct? Answers
a. 1 only 1. c
b. 2 only  ‗Saksham‘, an annual high intensity one-
month long people-centric mega campaign
c. Both 1 and 2 of Petroleum Conservation Research
d. Neither 1 nor 2 Association (PCRA) under the aegis of
Ministry of Petroleum and Natural Gas was
launched recently.
9) The selection of the Chairperson and the 2. b
members of Lokpal shall be through a Selection
Committee. Who among the following

www.shankariasacademy.com | www.iasparliament.com
 Unispace Nano satellite Assembly & Training  When people with Celiac disease eat gluten (a
programme (UNNATI), a capacity building protein found in wheat, rye and barley), their
programme on Nano-satellite body mounts an immune response
development, is an initiative by ISRO to that attacks the small intestine.
commemorate the 50th anniversary of the first
United Nations conference on the exploration  These attacks lead to damage on the villi,
and peaceful uses of outer space (UNISPACE- small fingerlike projections that line the small
50). intestine, that promote nutrient absorption.

 The programme provides opportunities to  When the villi get damaged, nutrients cannot
the participating developing be absorbed properly into the body and thus,
countries to strengthen in assembling, these patients fail to grow in height and
integrating and testing of Nanosatellite. weight, develop chronic diarrhea, anemia (low
hemoglobin), and weakness of bones.
3. b
 To create awareness about early diagnosis
 Under the Pradhan Mantri Rozgar Protsahan and management of the disease of which
Yojana (PMRPY) scheme, Government is Indians generally “have little or no
paying full employers‘ contribution of knowledge”, the International Symposium on
12% (towards Employees‘ Provident Fund and Wheat related Disorders (ISWD) 2019 was
Employees‘ Pension Scheme both), for a period organized in New Delhi by the Celiac Society
of 3 years in respect of new employees who of India (CSI) from January 10-13.
have been registered with the EPFO on or after
1st April 2016, with salary up to Rs. 15,000 per 7. a
month.  Haryana becomes the first state to launch
 It is being implemented by Ministry of Cyber Security portal and toll free number for
Labour and Employment through the cyber security and cyber alerts at the National
Employees‘ Provident Fund Summit on Digital Innovation and Cyber
Organization (EPFO). Security organized at Gurugram recently.

4. d 8. b

 Zakiullah Khan Committee was  Petroleum Conservation Research Association


constituted to review the Waqf Properties (PCRA) is a registered society set up under
Lease Rules, 2014. the aegis of Ministry of Petroleum &
Natural Gas, Government of India.
 It presented its report to Union Minister for
Minority Affairs recently.  As a non-profit organization, PCRA is a
national government agency engaged
5. d in promoting energy efficiency in various
sectors of economy.
 Palestine has recently assumed the
chairmanship of G77 (for the year  The Government of India has set up Bureau
2019), the global body‘s largest of Energy Efficiency (BEE) under the
intergovernmental Organisation of provision of the Energy Conservation Act,
developing countries including India. 2001.
 Egypt was the previous Chair of the Group of  It assist in developing policies and strategies
77 (G77) for the year 2018. with a thrust on self-regulation and market
principles with the primary objective of
 Palestine is a non-member Observer
reducing energy intensity of the Indian
State at the world body (U.N).
economy within the overall framework
6. b of the Energy Conservation Act, 2001.
 Gluten allergy, also known as Celiac disease is 9. c
an autoimmune disease that occurs
 The selection of the Chairperson and the
because of ingestion of a protein, called
members of Lokpal shall be through a
gluten.
Selection Committee consisting of

www.shankariasacademy.com | www.iasparliament.com
1. The Prime Minister, most active region, while zone I is the
least.
2. The Speaker of the Lok Sabha,
2. Bureau of Indian Standards classifies
3. The Leader of the Opposition in the Lok Sabha,
seismic zones in India.
4. The Chief Justice of India or a sitting Supreme
3. All of the Andaman and Nicobar Islands
Court Judge nominated by the Chief Justice of
lie in Zone V.
India and
Which of the statements given above are
5. An eminent jurist to be nominated by the
correct?
President of India on the basis of
recommendations of the first four members of a. 1 and 2 only
the selection committee.
b. 1 and 3 only
10. c
c. 2 and 3 only
 Cassini was one of the most ambitious efforts
d. None
ever mounted in planetary exploration.
 It is a joint endeavor of NASA, ESA (the
European Space Agency) and the Italian 3) Consider the following statements with
space agency (ASI), Cassini was a respect to Indian National Centre for Ocean
sophisticated robotic spacecraft sent to study Information Services (INCOIS)
Saturn and its complex system of rings and 1. It is an autonomous body works under
moons in unprecedented detail. the Ministry of Earth Sciences (MoES).
 Scientists, using data from the Cassini 2. In case of a tsunami, it alerts not only
spacecraft, have recently found evidence of the Indian authorities, but also its
methane rainfall on the north pole of Titan, the neighbours.
largest moon of Saturn.
Which of the statement(s) given above is/are
correct?
19-01-2019 a. 1 only
1) Consider the following statements with b. 2 only
respect to Young Scientist programme
c. Both 1 and 2
1. Under this programme, 3 students from
each state and UT will be selected and d. Neither 1 nor 2
given lectures and access to R&D labs
for one month period.
4) Consider the following statements
2. It was launched by the Ministry of
Human Resources and Development. 1. It was famously known as the gateway to
the Nubra and Shyok Valleys of Jammu
Which of the statement(s) given above is/are and Kashmir.
correct?
2. It is one of the highest motorable pass in
a. 1 only the world.
b. 2 only 3. The pass is strategically important to
c. Both 1 and 2 India as it is used to carry supplies to
the Siachen Glacier.
d. Neither 1 nor 2
Identify the correct pass that matches with the
above description
2) Consider the following statements with a. Banihal Pass
respect to Seismic Zones in India
b. Jelep La
1. India is classified in to five seismic
zones, in which Zone V is seismically the c. Khardung La

www.shankariasacademy.com | www.iasparliament.com
d. Shipki La 8) ―Project Udayak‖ sometimes seen in the
news recently, is associated with which of the
following?
5) ―Global Risks Report 2019‖ is recently
a. Border Roads Organisation (BRO)
published by which of the following
organisations? b. Indian Space Research Organisation (ISRO)
a. World Economic Forum c. Defence Research and Development
Organisation (DRDO)
b. World Bank
d. Indian Meteorological Organisation (IMD)
c. UN Environment Programme
d. UNFCCC
9) Recently, a report on ‗Outbreak Readiness
and Business Impact‘ was released by which of
6) Consider the following statements with the following organisations?
respect to Chilika Lake
a. World Economic Forum
1. The lake is currently listed under the
b. World Health Organisation
Montreux Record of Ramsar
Convention on Wetlands. c. World Bank
2. Irrawaddy dolphins were found in this d. International Monetary Fund
Lake.
3. Increasing presence of sea grass in this
10) Consider the following statements with
lake indicates its healthy condition.
respect to Inner line Permit (ILP)
Which of the statement(s) given above is/are
1. It is an official travel document issued
correct?
by the Government of India to allow
a. 3 only inward travel of an Indian citizen into a
protected area for a limited period.
b. 1 and 2 only
2. Currently, the Inner Line Permit (ILP)
c. 2 and 3 only
is operational in Arunachal Pradesh,
d. 1, 2 and 3 Mizoram and Nagaland.
Which of the statement(s) given above is/are
correct?
7) Consider the following statements with
respect to Diffo Bridge a. 1 only
1. It was constructed over the Diffo River b. 2 only
in Jammu and Kashmir.
c. Both 1 and 2
2. It would provide uninterrupted access
d. Neither 1 nor 2
between Sind valley and Chenab valley
region.
Which of the statement(s) given above is/are Answers
correct?
1. a
a. 1 only
 The Indian Space Research
b. 2 only Organisation (ISRO) has recently
c. Both 1 and 2 announced the Young Scientist programme for
the young minds who want to explore
d. Neither 1 nor 2 themselves in space research.
 Under the programme, three students from
each state and UT will be selected.

www.shankariasacademy.com | www.iasparliament.com
 They will be given lectures and they will  Chilika Lake is claimed to have 20% of India‘s
be given access to R&D labs and practical seagrass distribution, which plays a vital role
experience of building a small satellite. in oxygen production and absorption of carbon
dioxide and acts as a purifier in aquatic
 If the satellite is good, ISRO will launch it. ecology.
 This programme is similar to U.S. space  The lake was included in the Montreux Record
agency NASA‘s student outreach. (threatened list) in 1993 due to change in its
2. c ecological character by Ramsar Convention.
 Earthquake - prone areas of the country have  The lake was removed from the Montreux
been identified on the basis of scientific inputs record in 2002 (1st from Asia) by Ramsar
relating to seismicity, earthquakes occurred in convention due to successful restoration.
the past and tectonic setup of the region. 7. d
 Based on these inputs, Bureau of Indian  Defence minister has recently inaugurated the
Standards, has grouped the country 426.60 metre long Pre Stressed Concrete Box
into four seismic zones, viz. Zone II, III, Girder type bridge over Diffo river on Roing-
IV and V. Koron-Paya road in Arunachal Pradesh.
 Of these, Zone V is seismically the most active  It would provide uninterrupted access between
region, while zone II is the least. Dibang valley and Lohit valley region of
 Broadly, Zone - V comprises entire Eastern Arunachal Pradesh and an all-weather
northeastern India, parts of Jammu and Road to the troops deployed on the China
Kashmir, Himachal Pradesh, Uttaranchal, Border.
Rann of Kutch in Gujarat, part of North Bihar  Dedicating the bridge to the nation, the
and Andaman & Nicobar Islands. Defence minister praised the commendable
 A strong earthquake of magnitude 6 hit the work being done by Project Udayak,
seismically active Andaman and Nicobar Border Roads Organisation (BRO) in
Islands recently. this region.
3. c 8. a

 The INCOIS is an institute under the  In the eighties, the two erstwhile easternmost
Ministry of Earth Sciences (MoES) and it Projects Vartak and Sewak had five and four
has a network buoys and other apparatus to Border Roads Task Forces respectively.
check rise in the sea level in case of a tsunami.  It was felt that a new Project was required for
 It alerts not only the Indian authorities, better command and control.
but also its neighbours in case of a  On 23 May 1989, a new Project named
tsunami. ―Udayak ―was sanctioned.
 A strong earthquake of magnitude 6 hit the  The Project was to take on responsibility of
seismically active Andaman and Nicobar roads and other works in erstwhile
Islands recently. districts of Siang, Lohit, Dibang Valley and
4. c Tirap of Arunachal Pradesh and Mon,
Tuensang and Mokokchung districts of
5. a Nagaland.
6. c  Dedicating the Diffo bridge to the nation, the
 A stable Irrawaddy population, increase Defence minister praised the commendable
in seagrass spread and appearance of work being done by Project Udayak,
sponges are some of the interesting positive Border Roads Organisation (BRO) in
aspects brought to light during this year‘s this region.
annual monitoring at Chilika lagoon. 9. a

www.shankariasacademy.com | www.iasparliament.com
 In its recent report on ‗Outbreak Readiness Which of the statement(s) given above is/are
and Business Impact: Protecting Lives correct?
and Livelihoods across the Global
a. 1 only
Economy‘, Geneva-based World Economic
Forum (WEF) says that the global b. 2 only
businesses face huge risks from a new era of
c. Both 1 and 2
epidemics and pandemics that may cause an
average annual economic loss of $ 570 d. Neither 1 nor 2
billion to the global GDP, posing a threat
similar to that from climate change.
10. c 3) Consider the following statements
1. It is one of the tiger reserve governed
 To enter Arunachal Pradesh, Nagaland
under the Project Tiger.
or Mizoram, Indians from other states need
an inner line permit (ILP) under the provisions 2. It constitutes an important component
of the Bengal Eastern Frontier Regulation of a forest landscape comprising
(BEFR) Act, 1873. Nagarhole, Mudumalai and
Sathyamangalam.
3. Recently, Environment ministry has
21-01-2019 opposed a proposal to construct
1) Consider the following statements with elevated roads over this Tiger Reserve.
respect to Pravasi Bhartiya Diwas (PBD)-2019 Identify the Tiger reserve using the description
1. Pravasi Bharatiya Diwas (PBD) is given above
celebrated once in every two years to a. Wayanad Tiger reserve
strengthen the engagement of the
overseas Indian community with the b. Kalakad-Mundanthurai Tiger reserve
Government of India and reconnect c. Sharavati Tiger reserve
them with their roots.
d. Bandipur Tiger reserve
2. It is for the first time, the event is being
held in Varanasi.
3. The theme for this year event is "Role of 4) Consider the following statements
Indian Diaspora in building New India". 1. A foreigner cannot seek information
Which of the statement(s) given above is/are under RTI act.
correct? 2. Central Information Commission does
a. 2 only not have jurisdiction over State
Information Commission.
b. 1 and 2 only
3. A complaint or appeal can be filed in
c. 2 and 3 only Central Information Commission
d. 1, 2 and 3 against an order of a State Information
Commission.
Which of the statement(s) given above is/are
2) Consider the following statements with correct?
respect to NGO-DARPAN portal
a. 1 only
1. It is mandatory for all NGOs seeking
FCRA services to register under this b. 1 and 2 only
portal. c. 1 and 3 only
2. It is being maintained under the aegis of d. 1, 2 and 3
NITI Aayog.

www.shankariasacademy.com | www.iasparliament.com
5) Kohassa also known as White-Bellied Sea a. World Economic Forum (WEF)
Eagle – a large bird of prey, sometimes seen in
b. World Bank
the news recently, is endemic to
c. International Monetary Fund
a. Keibul Lamjao National Park
d. None of the above
b. Chilika Lake
c. Gulf of Mannar
10) Varthur Lake, sometimes seen in the news
d. Andaman and Nicobar Islands
recently, is located in which of the following
cities?
6) Consider the following statements a. Chennai
1. It is one of India‟s oldest port cities. b. Kolkata
2. It is located between the Bhogavo and c. Bengaluru
Sabarmati rivers near the Gulf of
d. Mumbai
Khambat.
3. The name of the site means “the mound
of the dead”. Answers
Identify the Indus Valley Civilisation site using 1. d
the description given above
 Pravasi Bharatiya Divas (PBD) is
a. Dholavira celebrated once in every two years to
b. Lothal strengthen the engagement of the overseas
Indian community with the Government of
c. Mohenjo-daro India and reconnect them with their roots.
d. Kalibangan  The 15th PBD Convention is being held on 21-
23 January 2019 in Varanasi, Uttar
Pradesh.
7) The term “Urban Heat Island” refers to
 The theme of PBD Convention 2019 is "Role
a. Islands formed due to volcanic activity of Indian Diaspora in building New
b. Warming up of Islands due to rapid India".
urbanisation
 PBD every year has been taking place on 9th
c. Urban area that is warmer than its January to mark to arrival of father of the
surrounding rural areas nation Mahatma Gandhi back to India from
South Africa in 1915 but this year it is taking
d. None of the above place on 21st to align it with Kumbh Mela and
Republic day so that diaspora can have a rich
experience of India's culture at Prayagraj and
8) “Formosa Strait” connects which of grandeur at Rajpath.
the following?
 It was also made a biennial event from an
a. South China Sea and East China Sea annual event earlier before 2015.
b. South China Sea and Philippine Sea 2. b
c. East China Sea and North China Sea
 In a relief to thousands of Non-government
d. East China Sea and Yellow Sea Organisations (NGOs), the Union Home
Ministry has recently relaxed the
mandatory requirement of getting NGOs
9) “Global Economy Watch report” is registered on a NITI-Aayog-run online
published by which of the following portal NGO-DARPAN to receive foreign
organisation? donations.

www.shankariasacademy.com | www.iasparliament.com
 Now, it is optional for all NGO seeking FCRA  Urban Heat Island – Urban area that is
services to register under the DARPAN portal. significantly warmer than its surrounding rural
areas.
 However, associations/NGOs, already
registered with the Darpan portal or those 8. a
required to register under the portal since they
 The Taiwan Strait or Formosa
receive subsidies/grants/contribution from
Strait separates the island
public funds, may still continue to quote their
of Taiwan from mainland China.
DARPAN identification number while seeking
Foreign Contribution (Regulation) Act (FCRA)  The strait is part of the South China
related services. Sea and connects to the East China
3. d Sea to the north.
9. d
 The Centre has recently opposed the elevated
road, proposed to come up in Bandipur  India is likely to cross the United Kingdom in
Tiger Reserve to overcome the night traffic the world‘s largest economy rankings in 2019,
ban from 9pm to 6am. Global Economy Watch report published
4. b by global consultancy firm PwC stated.

 As per section 3 of the RTI Act any Indian  PwC‘s Global Economy Watch monitors trends
citizen can seek information under the Act. and issues affecting the global economy and
details its latest projections for the leading
 Central Information Commission does nations.
not have jurisdiction over a State Information
10. c
Commission nor a complaint or appeal can be
filed in this Commission against an order of a  Spontaneous fire has been spotted recently
State Information Commission. on Bengaluru‘s Varthur Lake.
5. d
 To enhance the operational capability of 22-01-2019
Andaman and Nicobar Command (ANC),
Naval Air Station (NAS) Shibpur will be 1) Consider the following statements with
commissioned as INS Kohassa. respect to World Capitals of
Architecture initiative
 INS Kohassa has been named after a White –
Bellied Sea Eagle, which is a large bird of prey 1. It was jointly launched by UNESCO and
endemic to Andaman and Nicobar the International Union of Architects
Islands (ANI). (UIA) in 2018.

 INS Kohassa will become the third Naval Air 2. It is intended to become
Base in the Andaman and Nicobar islands after an international forum for debates
INS Utkrosh at Port Blair and INS Baaz at about pressing global challenges from
Campbell Bay. the perspectives of culture, cultural
heritage, urban planning and
6. b architecture.
 Lothal is located between the Bhogavo and 3. Rio de Janeiro was named as the first
Sabarmati rivers near the Gulf of city to receive the World Capitals of
Khambat. Architecture title for 2020.
 The word Lothal, like Mohenjo- Which of the statement(s) given above is/are
daro, means the mound of the dead. correct?
 Mohenjo-daro is an IVC site in the province a. 3 only
of Sindh, Pakistan. b. 1 and 2 only
7. c c. 2 and 3 only
d. 1, 2 and 3

www.shankariasacademy.com | www.iasparliament.com
3. The “yellow” in the name refers to the
Yellow forest in Sudan from where the
2) ―World Energy Outlook 2018‖ was
virus originates.
published by which of the following
Organizations? Which of the statement(s) given above is/are
correct?
a. International Atomic Energy Agency (IAEA)
a. 3 only
b. Organisation of the Petroleum Exporting
Countries (OPEC) b. 1 and 2 only
c. World Economic Forum (WEF) c. 1 and 3 only
d. International Energy Agency (IEA) d. 1, 2 and 3

3) ―Operation Kabaddi‖ sometimes seen in the 6) ―World Economic Outlook‖ is published by


news recently, is associated with which of the which of the following Organizations?
following?
a. World Bank
a. Indian Army
b. International Monetary Fund
b. Government of Kerala
c. World Economic Forum
c. National Disaster Response Force
d. None of the above
d. Government of Odisha

7) Consider the following statements with


4) Consider the following statements respect to Pulicat Lake
1. Indian citizens going to Nepal and 1. It is the second largest brackish water
Bhutan don‟t need a visa, if they have a ecosystem in the country after the
photo identity card issued by the Chilika Lake in Odisha.
government of India.
2. Flamingo festival will be held in this
2. All citizens of India can use Aadhaar lake every year.
card as a valid travel document to travel
Which of the statement(s) given above is/are
to Nepal and Bhutan.
correct?
Which of the statement(s) given above is/are
a. 1 only
correct?
b. 2 only
a. 1 only
c. Both 1 and 2
b. 2 only
d. Neither 1 nor 2
c. Both 1 and 2
d. Neither 1 nor 2
8) ―Rio Grande River‖, sometimes seen in the
news recently, forms part of the border
5) Consider the following statements with between which of the following two countries?
respect to Yellow Fever
a. Ireland and Northern Ireland
1. It is an acute viral hemorrhagic disease,
b. Brazil and Venezuela
transmitted by infected mosquitoes.
c. Norway and Sweden
2. The yellow fever virus is endemic in
tropical areas of Africa and Central and d. United States of America and Mexico
South America.

www.shankariasacademy.com | www.iasparliament.com
9) ―Warrior-VI‖ is a bilateral military exercise posts along the Line of Control (LoC) in
between which of the following two countries? September 2001.
a. China and Pakistan  The objective of Operation Kabaddi was to
change the geography of the LoC with access to
b. Russia and China
tactical points there, which would then help
c. Japan and U.S.A the Army tackle the infiltration of militants by
the Pakistani side.
d. Pakistan and Russia
 But it was not executed.

10) Consider the following pairs 4. a

1. Deopahar Reserve Forest – Assam  Indian citizens going to Nepal and Bhutan
don't need a visa, if they have a valid
2. Nelapattu Bird Sanctuary – Andhra passport, a photo identity card issued
Pradesh by the government of India or an
Which of the pair(s) given above is/are election ID card issued by the Election
correctly matched? Commission.

a. 1 only  Aadhaar cards are now valid travel document


for Indians under 15 and over 65 travelling
b. 2 only to Nepal and Bhutan, a recent communique
c. Both 1 and 2 by the MHA said.

d. Neither 1 nor 2  Indians, other than those in the two age


brackets will not be able to use Aadhaar to
travel to the two neighbouring countries.
Answers 5. b
1. d  Yellow fever is often associated
with jaundice, hence the name yellow.
 The UN Educational, Scientific, and Cultural
Organization (UNESCO) has named the 6. b
Brazilian city of Rio de Janeiro as the
World Capital of Architecture for 2020.  World Economic Outlook is published
by International Monetary Fund (IMF),
 Rio will be the first city to receive the title biannually and partly updated two times a
under a programme launched together by year.
UNESCO and the International Union of
Architects (UIA) in November 2018.  The IMF in its recent January World Economy
Outlook update said, India would remain the
2. d fastest growing major economies of the world.
 The World Energy Outlook is published  It projected that India will grow at 7.5 per cent
by International Energy Agency (IEA). this year and 7.7 per cent in 2020, an
impressive over one percentage point ahead of
 It is published every year, based on objective China's estimated growth of 6.2 per cent in
data and dispassionate analysis.
these two years.
 It provides critical analysis and insights on 7. c
trends in energy demand and supply, and what
they mean for energy security, environmental  Flamingo festival is an annual three-day
protection and economic development. festival held in the Pulicat Lake.
3. a  The Flamingo Festival has recently kicked-off
at Pulicat Lake and Nellapattu in Sullurpet
 Operation Kabaddi was proposed to be a mandal, Andhra Pradesh.
largest operation to end cross-LoC infiltration
by capturing at least 25 selected Pakistani 8. d

www.shankariasacademy.com | www.iasparliament.com
 The River Rio Grande is one of the 1. It usually lives in deep, coastal waters.
principal rivers in the southwest United
2. It is known to occur within the Indian
States and northern Mexico.
Ocean from South Africa to India.
 The Rio Grande begins in south- 3. It has been currently categorised as
central Colorado in the United States and flows Endangered by the IUCN.
to the Gulf of Mexico.
Which of the statements given above are
 Along the way, it forms part of correct?
the Mexico–United States border.
a. 1 and 2 only
 Recently, U.S President visit the Rio Grande
Valley to gather support for his U.S-Mexico b. 1 and 3 only
border wall plan. c. 2 and 3 only
9. a d. 1, 2 and 3
 The Pakistan-China joint military
exercise ‗Warrior -VI‘ was recently
concluded near Kharian, Gujrat District of the 3) ―Global Commission report on the Future of
Punjab province of Pakistan. Work‖ was recently published by which of the
following organisations?
10. c
a. International Labour Organisation (ILO)
 The Supreme Court has recently dismissed an
appeal filed by Numaligarh Refinery Ltd. b. United Nations (UN)
(NRL) and ordered demolition of a 2.2-km c. World Economic Forum (WEF)
boundary wall erected on an elephant
migration corridor in the Deopahar protected d. World Bank
Reserve Forest in eastern Assam‘s Golaghat
district.
4) The area known as ‗Golan
Heights‘ sometimes appears in the news in the
context of the events related to
23-01-2019
a. Central Asia
1) Consider the following statements with
respect to Sea Vigil – 2019 b. Middle East
1. It is a bilateral naval exercise between c. South-East Asia
India and Mauritius.
d. Central Africa
2. The aim of this exercise is to have an
effective forum to discuss common
concerns in the Indian Ocean Region 5) Consider the following statements with
and forge deeper cooperation among respect to Malabar exercise
friendly navies.
1. It began as a bilateral naval exercise
Which of the statement(s) given above is/are between India and the US and later
correct? expanded into a trilateral format with
the inclusion of Japan.
a. 1 only
2. Recently, Australia was included in the
b. 2 only
exercise.
c. Both 1 and 2
Which of the statement(s) given above is/are
d. Neither 1 nor 2 correct?
a. 1 only
2) Consider the following statements with b. 2 only
respect to Indian Ocean Humpback Dolphin
c. Both 1 and 2

www.shankariasacademy.com | www.iasparliament.com
d. Neither 1 nor 2 9) Consider the following statements with
respect to AMRIT Scheme
1. It aims to provide quality antenatal
6) Consider the following statements with
care, free of cost, universally to all
respect to Bharat Parv – 2019
pregnant women on the 9th of every
1. It is being celebrated as part of Pravasi month.
Bharatiya Divas (PBD) celebrations
2. It is an initiative of Ministry of Health
since 2016.
and Family Welfare.
2. It is being organised by the Ministry of
Which of the statement(s) given above is/are
Tourism.
correct?
Which of the statement(s) given above is/are
a. 1 only
correct?
b. 2 only
a. 1 only
c. Both 1 and 2
b. 2 only
d. Neither 1 nor 2
c. Both 1 and 2
d. Neither 1 nor 2
10) ―Kani tribes‖ sometimes seen in the news
recently inhabits which of the following states?
7) Consider the following statements with
a. Tamilnadu
respect to Pradhan Mantri Jan Aushadhi
Yojana b. Goa
1. It aims to provide expensive branded c. Kerala
drugs to all at an affordable prices
d. Odisha
through Jan Aushadhi Stores.
2. It is a direct market intervention
scheme of Ministry of Health and family Answers
welfare.
1. d
Which of the statement(s) given above is/are
correct?  The Indian Navy, in close coordination with
the Indian Coast Guard and all coastal
a. 1 only security stakeholders, will conduct a
b. 2 only Coastal Defence Exercise code-named ‗Sea
Vigil 2019‘ along the entire coastline of the
c. Both 1 and 2 country.
d. Neither 1 nor 2  It is a large-scale exercise being conducted for
the first time ever and will aim to test
India‘s preparedness to thwart any attempt
8) ―Border by anti-national elements to carry out attack
Backstop Arrangement‖ sometimes seen in the on its territory or against its citizens by
news recently, is associated with which of the infiltrating through the sea route.
following?
2. c
a. U.S-Mexico wall
 The Indian Ocean Humpback Dolphin is
b. BREXIT known to occur within the Indian Ocean
c. U.S troops withdrawal from Syria from South Africa to India.
d. None of the above  The habitat preference of the dolphins
for shallow waters places them in some of
the world‘s most intensively utilised, fished,
shipped, modified and polluted waters, as per

www.shankariasacademy.com | www.iasparliament.com
the IUCN (International Union for  Malabar began as a bilateral naval exercise
Conservation of Nature). between India and the US in 1992 and has
grown in scope and complexity in recent years.
 Indian Ocean Humpback Dolphins are
concentrated in coastal waters within 2  In 2015, it was expanded into a trilateral
km of shore and they are often sighted only a format with the inclusion of Japan.
few hundred meters from land.
 However, India has not accepted
 This distribution places them in exactly the repeated requests from Australia to be
same location as the majority of small-scale included in the exercise, which is seen with
fishing effort prevalent throughout their range suspicion by China.
in the same near-shore habitat.
6. b
 As a result, humpback dolphins encounter
large numbers of coastal gillnets and are at  The Ministry of Tourism in collaboration
high risk of entanglement. with other central Ministries is organizing the
4th edition of ‗Bharat Parv‘ event showcasing
 IUCN currently categorises the Indian Ocean the spirit of Ek Bharat Shreshtha Bharat at
Humpback Dolphin as Endangered. Red Fort in the capital during 26th to
31stJanuary 2019.
3. a
 It is being celebrated as part of Republic
 The International Labour Organisation
Day Celebrations since 2016.
(ILO) has recently launched its Global
Commission report on the ‗Future of Work‘ to 7. d
mark its centenary celebration.
 Pradhan Mantri Jan Aushadhi Yojana is a
 The report is the result of a 15-month review direct market intervention scheme of the
by a 27-member commission co-chaired by the Department of Pharmaceuticals, Ministry of
South African president Cyril Ramaphosa and Chemicals and Fertilisers.
the Swedish Prime Minister Stefan Lofven.
 Its objective is to make available quality
 A universal labour guarantee, social protection generic medicines at affordable prices
from birth to old age and an entitlement to to all, especially the poor, throughout the
lifelong learning are among the ten country, through outlets known as Jan
recommendations made in the report. Aushadhi Stores (JASs).
4. b  Taking inspiration from the success that Jan
Aushadhi have had with private franchisees,
 The Golan Heights is located on the border
the Health Ministry has decided to throw open
between Israel and Syria.
AMRIT stores to private players in a bid to
 Today it‘s mostly occupied by Israel, but it was increase access to affordable drugs.
under Syrian sovereignty until June 1967, 8. b
when Israel took control of most of the area in
the Six-Day War, a military conflict with 9. b
Egypt, Jordan and Syria.
 The Affordable Medicines and Reliable
 Although Israel controls only about two-thirds Implants for Treatment (AMRIT) pharmacies
of what is known as the Golan Heights, it is a scheme aims to provide expensive drugs
controls the most strategic points. such as those used for treating cancer and
cardiovascular diseases at a cost that is 30-40
 Recently, Israel has carried out a series of per cent cheaper.
airstrikes against Iranian targets in Syria, in
response to an Iranian missile fired at the  Unlike Jan Aushadhi stores, which are run by
Golan Heights. the Ministry of Chemicals and
Fertilisers, AMRIT stores do not sell
5. a generic versions of drug but sell
branded drugs.

www.shankariasacademy.com | www.iasparliament.com
 Taking inspiration from the success that Jan 2. It was first built during the reign of
Aushadhi have had with private franchisees, Krishnadevaraya.
the Health Ministry has decided to throw open
3. It has features of both Nagara and
AMRIT stores to private players in a bid to
Dravida Styles.
increase access to affordable drugs.
Which of the above statement(s) is/are
10. c
correct?
 The Kanis, once a nomadic tribe that a. 1 only
subsisted on forest produce, including
medicinal plants, are now settled on forest b. 2 only
land in the southern districts of c. 2 and 3 only
Thiruvananthapuram and Kollam of Kerala.
d. All of the above
 The Kanis‘ worship an idol of Agastya Muni on
the hill and, according to them, permitting
women beyond Athiramala, a base station of 4) Consider the following statements
Agasthyakoodam, would interfere with their about Section 124A of IPC
traditional rights of worship.
1. It is unconstitutional as it violates
fundamental right of freedom of speech
24-01-2019 & expression.

1) Consider the following statements 2. It was introduced during the period of


about Caesium-137 Lord Lytton.

1. It is the only stable isotope of Caesium. Which of the above statement(s) is/are
correct?
2. It exists in liquid form at room
temperature. a. 1 only

3. It is a part of the spent fuel in nuclear b. 2 only


plants. c. Both 1 and 2
Which of the above statement(s) is/are d. Neither 1 nor 2
correct?
a. 1 only
5) Consider the following statements about
b. 2 only National Family Health survey (NFHS).
c. 2 and 3 only 1. It is a survey undertaken by the Ministry
d. All of the above of statistics and programme
implementation.
2. International Institute for Population
2) Hukitola Island is situated in the coast of Sciences (IIPS) is the nodal agency
a. Karnataka responsible for providing coordination
and technical guidance.
b. Kerala
Which of the above statement(s) is/are
c. Andhra Pradesh correct?
d. Odisha a. 1 only
b. 2 only
3) Consider the following statements c. Both 1 and 2
about Vittala Temple
d. Neither 1 nor 2
1. It is dedicated to Lord Vishnu.

www.shankariasacademy.com | www.iasparliament.com
6) Consider the following statements regarding 2. Elevation within 700m above mean sea
National Bench of Goods and Services Tax level
Appellate Tribunal (GSTAT)
3. Red sandy loam soil with proper
1. It is the forum of first appeal in GST drainage.
laws.
4. Moderate rainfall with no frost
2. It is the first common forum of dispute
The above conditions are necessary for which
resolution between Centre and States.
of the following crops?
Which of the above statement(s) is/are
a. Tea
correct?
b. Cashew
a. 1 only
c. Jute
b. 2 only
d. Cotton
c. Both 1 and 2
d. Neither 1 nor 2
10) Consider the following statements
about Western Disturbances
7) The report 'Hidden Risks and Untapped
1. It originates to the west of Iran.
Opportunities: Water and the Indian Banking
Sector' was launched by _______ along with 2. It picks up moisture from the Atlantic
Indian Banking Association (IBA) Ocean.
a. World Wildlife Fund-India 3. The brings rainfall to Northwestern
India, Gangetic plains and Northeast
b. World Economic Forum
India.
c. World Bank
4. It brings rains,
d. UNDP thunderstorms, hailstorm and snowfall.
Which of the above statement(s) is/are
correct?
8) Consider the following statements regarding
various development indicators. a. 1 and 2 only
1. Global Competitiveness Report - b. 1, 2 and 3 only
World Economic Forum
c. 1, 3 and 4 only
2. Global Talent Competitiveness Index -
d. All of the above
INSEAD
3. Global Democracy Index -
Economic Intelligence Unit (EIU) Answers
Which of the above Report(s)/Index are 1. c
correctly matched?
 Cs-133 is the most stable isotope of Caesium
(Cs)
a. 1 only
 Cs-137 decays through the emission of beta
b. 2 only particles (a high-energy electron or positron,
c. 3 only or positive electron) and gamma rays (a form
of electromagnetic radiation like X-rays).
d. All of the above
 It has a half-life of about 30 years.
 It is a byproduct in fission reactions of
9) Consider the following conditions
uranium and plutonium in nuclear plants.
1. Temperature of 20 to 30 degree Celcius

www.shankariasacademy.com | www.iasparliament.com
 Cs-137 is used in a variety of measuring  But it is the first common forum of dispute
instruments in the construction and other resolution between Centre and States.
industry.
7. a
 It is also used in well-logging devices in the
8. d
drilling industry for the characterisation of
rocks. 9. b
2. d 10. d
3. a 1. It originates beyond Afghanistan and Iran.
 It was built during the reign of King Devaraya 2. It picks up moisture from as far as the
II (1422 – 1446 A.D.). Mediterranean Sea, and even the Atlantic
Ocean.
 Several portions of the temple were expanded
and enhanced during the reign 3. The winds travel in the form of storms from
of Krishnadevaraya (1509 – 1529 A.D.). the northwestern direction and cause frequent
rainfall over Pakistan, North and
 It is built in the Dravidian style of architecture. Northwestern India, Uttarakhand, and some
It has traits and features that are characteristic parts of the Gangetic plains. Sometimes the
of typical south Indian temple architecture. disturbances reach even the Northeast of
 On the other hand Badami caves of Chalukya India.
Dynasty simultaneously represent two ancient 4. It brings all kinds of preciptation, including
Indian artistic traditions; the rain and thunderstorms, hailstorm
northern Nagara and the and snowfall depending upon the temperature
southern Dravida styles. of the region.
4. d
 The ‗public order‘ clause, one of the 25-01-2019
‗reasonable restrictions‘, allows freedom of
speech to be abridged by a law. 1) Consider the following statements with
respect to Rashtriya Bal Puraskar awards
 Sedition was introduced in the penal code in
1870, which was the period of Lord Mayo, a 1. It has been given in two categories: Bal
decade after the Indian Penal Code came into Kalyan Puraskar to individuals and Bal
force. Shakti Puraskar to institutions working
for children.
5. b
2. It has been introduced by the Ministry
 The National Family Health Survey (NFHS) is of women and child development.
a large-scale, multi-round survey conducted in
a representative sample of households Which of the statement(s) given above is/are
throughout India under the stewardship of correct?
the Ministry of Health and Family a. 1 only
Welfare.
b. 2 only
6. b
c. Both 1 and 2
 The way that GST was envisaged that
the assessment would be done by people below d. Neither 1 nor 2
the rank of Commissioner.
 First appeals are dealt by the Appellate 2) Consider the following statements with
Authorities (of Commissioner rank) under the respect to Long Range Surface-to-Air Missile
Central and State GST Acts (LRSAM)
 GSTAT woulde the forum of second appeal in 1. It is an indigenous missile designed and
GST laws developed by DRDO.

www.shankariasacademy.com | www.iasparliament.com
2. Having an operational range of 1500
km, it can destroy airborne threats like
6) Consider the following statements with
aircrafts, anti-ship missiles, UAVs and
respect to National Voters‟ Day (NVD)
rockets launched simultaneously.
1. It is celebrated all over the country on
Which of the statement(s) given above is/are
January 25 every year to mark the first
correct?
election held in India after
a. 1 only Independence.
b. 2 only 2. It was celebrated since 2011 to
encourage, facilitate and maximize the
c. Both 1 and 2
enrolment, especially for the new
d. Neither 1 nor 2 voters.
Which of the statement(s) given above is/are
correct?
3) Consider the following statements with
respect to PSLV C-44 mission a. 1 only
1. It is the first flight with a new variant of b. 2 only
PSLV called PSLV-DL with no strap-ons
c. Both 1 and 2
(boosters).
d. Neither 1 nor 2
2. Microsat-R satellite, developed by the
students was launched through this
mission.
7) Consider the following statements with
Which of the statement(s) given above is/are respect to Zaid Season
correct?
1. It is a short season between the rabi and
a. 1 only the kharif seasons.
b. 2 only 2. Water melon, Musk melon and
Cucumber are some of the Zaid crops.
c. Both 1 and 2
Which of the statement(s) given above is/are
d. Neither 1 nor 2
correct?
a. 1 only
4) ―Global Multi-dimensional Poverty Index
b. 2 only
(MPI) 2018 Report‖ was recently released
jointly by the Oxford Poverty and Human c. Both 1 and 2
Development Initiative (OPHI) and _______
d. Neither 1 nor 2
a. World Bank
b. Oxfam
8) Consider the following statements with
c. UNDP respect to New Generation Anti-Radiation
Missile (NGARM)
d. None of the above
1. It is an indigenous surface-to-surface
missile system.
5) ―My Vote Matters‖ which was launched by
2. It is capable of destroying enemy
the Election Commission of India recently, is
radars, tracking systems and
a/an
communication facilities.
a. Online Information repository
Which of the statement(s) given above is/are
b. Door to Door campaign correct?
c. Magazine a. 1 only
d. None of the above b. 2 only

www.shankariasacademy.com | www.iasparliament.com
c. Both 1 and 2 king for children. (erstwhile known as
National Child Welfare Award).
d. Neither 1 nor 2
2. d
 LRSAM has been jointly developed
9) ―Kerch Strait‖ sometimes seen in the news
by Defence Research and Development
recently, separates which of the following?
Organisation (DRDO), India and
a. Black Sea and the Sea of Marmara M/s IsraelAerospace Industries (IAI), Israel
for the Indian Navy.
b. Gulf of Mexico and Caribbean Sea
c. Sea of Azov and the Black Sea  The missile having an operational range of 75
km can destroy airborne threats like jets,
d. None of the above aircraft, anti-ship missiles, unmanned aerial
vehicles and rockets including projectiles
launched simultaneously.
10) Consider the following statements with
3. d
respect to World Economic Forum (WEF)
1. It is an International Organization for  The PSLV-C44, was the first mission of a new
Public-Private Cooperation. variant of the PSLV, called the PSLV-DL,
as it was equipped with two strap-
2. Its headquarters is located in Geneva, onconfigurations.
Switzerland.
 Usually, PSLVs were launched without any
3. Global gender gap report and Global strap-ons (boosters) or were equipped with six
competitiveness report are published by strap-ons fixed around the rocket, but the
the WEF. ISRO, for the first time, used only two boosters
Which of the statement(s) given above is/are for this mission.
correct?  India successfully launched Microsat-R, an
a. 2 only imaging satellite, meant for military
purposes developed by DRDO
b. 1 and 2 only laboratories using PSLV-C44.
c. 2 and 3 only  Contributed by students and the members of
d. 1, 2 and 3 a Chennai-based organisation — Space Kidz
India — Kalamsat is also launched using this
mission.
Answers 4. c
1. b 5. c
 The Ministry of Women and Child  ECI has released ‗My Vote Matters‘, a
Development, Government of India has Quarterly Magazine on the occasion of
introduced a revamped award scheme under National Voters Day 2019 celebration.
the name of ―Pradhan Mantri National
6. b
Children Awards‖ (Pradhan Mantri
Rashtriya Bal Puraskar).  The National Voters‘ Day (NVD) is celebrated
all over the country on January 25 every
 These awards will be given in two main
year since 2011 to mark the Foundation
categories. The categories are as follows:
day of Election Commission of India,
1. Bal Shakti which was established on 25th January 1950.
Puraskar to individuals (erstwhile known
as National Child Award)  The main purpose of the NVD celebration is to
encourage, facilitate and maximize the
2. Bal Kalyan enrolment, especially for the new voters.
Puraskar to institutions/individuals wor
 In view of the upcoming Lok Sabha
Elections, ―No Voter to be Left

www.shankariasacademy.com | www.iasparliament.com
Behind‖ has been selected as the theme for 4. Emission from livestock manure
the National Voters‘ Day, 2019.
Select the correct answer using the code given
7. c below
 In between the rabi and the kharif seasons, a. 2 only
there is a short season during the summer
b. 1 and 4 only
months known as the Zaid season.
c. 2 and 4 only
 Some of the crops produced during ‗zaid‘ are
watermelon, muskmelon, cucumber, d. 1, 2 and 3 only
vegetables and fodder crops.
 Ministry of Agriculture & Farmers Welfare has 2) Consider the following statements with
recently organized the national conference on respect to Bharat Ratna
agriculture for Zaid/ Summer campaign-2019.
1. The award does not carry any monetary
8. b grant.
 NGARM, having a strike range of over 100 km, 2. It cannot be used as a prefix or suffix to
is the first air-to-surface the recipient‟s name.
missile completely designed and
developed by DRDO. 3. It can be awarded posthumously.

 It is the first indigenous air to surface missile 4. The recommendations for the Bharat
to be developed by DRDO, after the supersonic Ratna are made by the Prime Minister
Brahmos cruise missile developed jointly with to the President.
Russia. Which of the statements given above are
 The missile is capable of destroying enemy correct?
radars, tracking systems and communication a. 1 and 3 only
facilities.
b. 2 and 4 only
 The state-of-the-art high speed missile
is first-of-its-kind in the country‘s c. 1, 2, 3 and 4
arsenal. d. None
 Recently the DRDO claimed to have conducted
the maiden test of new generation anti-
radiation missile (NGARM) from a fighter 3) Consider the following statements with
aircraft off Odisha coast. respect to Mugger Crocodile

9. d 1. It is endemic to freshwater habitats of


Gujarat.
 The Kerch Strait is a strait connecting the
2. It has been listed as endangered on the
Black Sea and the Sea of Azov, separating
IUCN Red List.
the Kerch Peninsula of Crimea from the
Taman Peninsula of Russia. 3. It is protected under Schedule I of the
Wildlife Protection Act, 1972.
10. d
Which of the statement(s) given above is/are
correct?
26-01-2019 a. 3 only
1) Which of the following is/are not the b. 1 and 2 only
cause(s) of Nitrogen Pollution
c. 1 and 3 only
1. Emission from chemical fertilizers
d. 1, 2 and 3
2. Oil spillage in oceans
3. Burning of fossil fuels

www.shankariasacademy.com | www.iasparliament.com
4) Consider the following statements 1. The act extends to the whole of India
with the objective of effectively
1. Ashoka Chakra is India's highest
protecting the wild life of India.
military decoration, awarded for
displaying distinguished acts of valour 2. Among the six schedules listed in the
during wartime. Act, Schedule I and part II of Schedule II
provide the highest degrees of
2. Kirti Chakra is India's second highest
protection to listed species.
peacetime gallantry award.
Which of the statement(s) given above is/are
Which of the statement(s) given above is/are
correct?
correct?
a. 1 only
a. 1 only
b. 2 only
b. 2 only
c. Both 1 and 2
c. Both 1 and 2
d. Neither 1 nor 2
d. Neither 1 nor 2

8) ―World Economic Situation and Prospects


5) ―Red fort declaration‖ sometimes seen in
Report 2019‖ was recently launched by which
the news recently, established a Strategic
of the following organizations?
Partnership between India and ________
a. United Nations (UN)
a. Mauritius
b. Federal Reserve Bank of USA
b. South Africa
c. World Economic Forum
c. Japan
d. International Monetary Fund
d. Seychelles

9) Carbon Offsetting and Reduction Scheme for


6) Which of the following countries does
International Aviation (CORSIA) was
not share border with Venezuela?
developed by
1. Peru
a. International Air Transport Association
2. Guyana (IATA)
3. Colombia b. International Civil Aviation Organization
(ICAO)
4. Suriname
c. International Aviation Regulation Authority
5. Brazil (IARA)
Select the correct answer using the code given d. None of the above
below
a. 1 and 4 only
10) Consider the following statements with
b. 2 and 4 only respect to Beti Bachao Beti Padhao (BBBP)
c. 2, 3 and 5 only Scheme
d. 1, 2, 4 and 5 only 1. It is a Central Sector Scheme with 100%
financial assistance for District level
component.
7) Consider the following statements with 2. It is a tri-ministerial effort of Ministries
respect to Wild life protection Act, 1972 of Women and Child Development,
Health & Family Welfare and Human
Resource Development.

www.shankariasacademy.com | www.iasparliament.com
Which of the statement(s) given above is/are  The award was also conferred on RSS
correct? ideologue Nanaji Deshmukh and singer
Bhupen Hazarika (both posthumously).
a. 1 only
3. a
b. 2 only
c. Both 1 and 2  The mugger crocodile, also called marsh
crocodile or broad-snouted crocodile, is a
d. Neither 1 nor 2 species (Crocodylus palustris) native to
freshwater habitats from southern Iran and
Pakistan to the Indian subcontinent and Sri
Answers Lanka.
1. a  It has come to the centre of renewed attention
in Gujarat, where the Forest Department has
2. c started evacuating muggers from two ponds on
 Bharat Ratna, the country‘s highest civilian the Sardar Sarovar Dam premises on the
award instituted in 1954, is given in Narmada, to facilitate a seaplane service at the
recognition of exceptional service, Statue of Unity.
performance of the highest order in any field of  Already extinct in Bhutan and Myanmar, the
human endeavour. mugger has been listed as vulnerable on the
 Any person without distinction of race, IUCN Red List since 1982.
occupation, position or sex is eligible for this  In India, it is protected under Schedule I of the
award. Wildlife Protection Act, 1972.
 The recommendations for Bharat Ratna are 4. b
made by the Prime Minister to the
President.  Ashok Chakra is the
highest peacetime award given for valor and
 The number of annual awards is restricted to a bravery away from the face of the enemy and it
maximum of three in a particular year. is equivalent to Param Veer Chakra, which
 On conferment of the award, the recipient is the highest war-time gallantry award.
receives a Sanad (certificate) signed by the  Kirti Chakra is the second in the order of
President and a medallion. precedence of the peacetime gallantry award
 The award does not carry any and is equivalent to Maha Veer
monetary grant. chakra, the second highest military
decoration, given to the act of braveness
 The award cannot be used as a prefix or performed during war.
suffix to the recipient‘s name.
5. b
 However, should an award winner consider it
necessary, he or she may use the following  The Strategic Partnership between India and
expression in their biodata or letterhead or South Africa, called the Red Fort
visiting card etc. to indicate that he or she is a Declaration, was signed in March 1997 by the
recipient of the award: ‗Awarded Bharat Ratna then South African President Nelson Mandela
by the President‘ or ‗Recipient of Bharat Ratna and former PM H.D. Deve Gowda.
Award‘.  India and South Africa has recently sealed a
 The original statutes did not provide for three-year strategic programme covering all
posthumous awards but were amended aspects of bilateral ties.
in January 1955 to permit them. 6. a
 The President has recently conferred Bharat  Venezuela is located on the northern coast of
Ratna, on former president Pranab South America.
Mukherjee.
 The continental territory is bordered on
the north by the Caribbean Sea and

www.shankariasacademy.com | www.iasparliament.com
the Atlantic Ocean, on the west by  Given the advent of Carbon Offsetting and
Colombia, Brazil on the south, Trinidad Reduction Scheme for International Aviation
and Tobago to the north-east and on (CORSIA) by the International Civil Aviation
the east by Guyana. Organisation (ICAO) by 2027, this is a
significant development which could reduce
7. b
the carbon emissions and help India become a
 It extends to the whole of India, except the green fuel production hub.
State of Jammu and Kashmir which has its
10. c
own wildlife act.
 Beti Bachao Beti Padhao (BBBP) is
 It has six schedules which give varying degrees
a comprehensive programme to address the
of protection
declining Child Sex Ratio (CSR) and related
 Species listed in Schedule I and part II of issues of empowerment of women over a life-
Schedule II get absolute protection — offences cycle continuum.
under these are prescribed the highest
 It is a tri-ministerial effort of Ministries of
penalties
Women and Child Development, Health &
 Species listed in Schedule III and Schedule IV Family Welfare and Human Resource
are also protected, but the penalties are much Development.
lower
 BBBP Scheme is a Central Sector Scheme
 Schedule V includes the animals which may be with 100% financial assistance for
hunted District level component and the fund are
directly released to the DC/DM‘s account for
 The plants in Schedule VI are prohibited from smooth operation of the Scheme.
cultivation and planting
 The specific objectives of the scheme include
8. a preventing gender biased sex selective
 The Indian economy is expected to grow at 7.4 elimination; ensuring survival and protection
per cent in the current fiscal year and its GDP of the girl child and ensuring education and
growth will rise to 7.6 per cent in 2019-20, participation of the girl child.
before tapering down to 7.4 per cent in 2020-
21, according to a UN report: World
Economic Situation and Prospects 28-01-2019
(WESP) 2019, released recently.
1) Consider the following statements
9. b
1. It is located in Western Indian Island
 Carbon Offsetting and Reduction Scheme for which is also known as the Island of
International Aviation (CORSIA), is an Gharapuri.
emission mitigation approach for the global 2. It is a UNESCO World Heritage site.
airline industry, developed by
the International Civil Aviation 3. It is a glorious abode of Lord Shiva and
Organization (ICAO). an epitome of Hindu cave culture.
 The aim is to address any annual increase in Identify the world heritage site using the
total CO2 emissions from international civil description given above
aviation above 2020 levels and contribute to a. Ellora Caves
the industry‘s commitment to carbon neutral
growth from 2020 (―CNG2020‖). b. Bandar Khayran Caves
 Recently, the Bureau of Indian Standards in c. Elephanta Caves
Collaboration with the Indian Air Force d. Silvassa Island Temple
Releases New Standard for Bio-Jet Fuel.
 This standard would enable the oil companies
to manufacture bio-jet fuel for the Indian 2) Consider the following statements with
aviation industry. respect to Pariksha Pe Charcha 2.0 programme

www.shankariasacademy.com | www.iasparliament.com
1. It is a unique interaction platform, d. None
where students can directly register
their grievances with respect to exams,
with the Human Resource Minister. 5) Consider the following statements with
respect to Serious Fraud Investigation Office
2. Only Delhi / NCR students were allowed
(SFIO)
to participate in this programme.
1. It was set up by a resolution adopted by
Which of the statement(s) given above is/are
the Ministry of Corporate Affairs in
correct?
2013.
a. 1 only
2. It is a multi-disciplinary organization
b. 2 only works under the Ministry of Finance.
c. Both 1 and 2 Which of the statement(s) given above is/are
correct?
d. Neither 1 nor 2
a. 1 only
b. 2 only
3) Consider the following statements with
respect to Technical textiles c. Both 1 and 2
1. They are manufactured primarily for d. Neither 1 nor 2
technical performance and functional
properties rather than aesthetic and
decorative characteristics. 6) DAMaN initiative, an inspiration for global
fight against malaria, is an effort by which of
2. Use of technical textiles is restricted
the following Indian states?
only to medical applications in India.
a. Chattisgarh
Which of the statement(s) given above is/are
correct? b. Odisha
a. 1 only c. Meghalaya
b. 2 only d. West Bengal
c. Both 1 and 2
d. Neither 1 nor 2 7) Consider the following statements with
respect to National Chilika Bird Festival
1. It aims to promote eco-tourism and
4) Consider the following statements with
preservation of birds in India.
respect to Animal Welfare Board of India
(AWBI) 2. It was jointly organised by the Ministry
of Tourism and Odisha Tourism
1. It was set up by a resolution of the
Development Board.
Ministry of Environment, Forest and
Climate Change. Which of the statement(s) given above is/are
correct?
2. It is headed by the Prime Minister of
India. a. 1 only
3. Its headquarters is located in Chennai, b. 2 only
Tamilnadu.
c. Both 1 and 2
Which of the statement(s) given above is/are
correct? d. Neither 1 nor 2

a. 3 only
b. 1 and 3 only 8) In which of the following states, Schedule VI
of Indian Constitution does not apply?
c. 1, 2 and 3

www.shankariasacademy.com | www.iasparliament.com
1. Assam  The masterpiece ―Sadashiva‖ dominates the
entrance to one of the caves in Elephanta.
2. Arunachal Pradesh
3. Meghalaya  A unique event “Artisan Speak” which will
showcase the rich handloom and textile
4. Manipur tradition of India is being held at Elephanta
5. Nagaland Caves recently.

6. Mizoram  The first-of-its-kind event at Elephanta Caves


is being organized by the Ministry of Textiles,
Select the correct answer using the code given in partnership with IMG Reliance.
below
2. d
a. 2 and 6 only
 It is a unique interaction where students,
b. 1, 3, and 6 only teachers, parents and the Prime
c. 2, 3 and 5 only Minister, will come together to discuss
issuesrelated to exams and the stress
d. 2, 4 and 5 only connected to that.
 A total of 2000 students, parents and teachers
9) ―Nalbana Island‖ is located in which of the will be participating at Talkatora Stadium
following states? from all over India in another exciting edition
of Pariksha Pe Charcha 2.0 on 29th
a. Maharashtra January, 2019.
b. Karnataka  For first time, students from all over
c. Kerala India and also Indian students residing
abroad are participating as compared to last
d. Odisha year where only Delhi / NCR students were
called.

10) Consider the following pairs  The participants from abroad include from
countries like Russia, Nigeria, Iran, Nepal,
1. Satkosia Tiger Reserve – West Bengal Doha, Kuwait, Saudi Arabia, and Singapore.
2. Simlipal Tiger Reserve – Odisha 3. a
3. Cauvery Wildlife Sanctuary – Karnataka  Technical textiles are textile material and
Which of the Pair(s) given above is/are products manufactured primarily for technical
correctly matched? performance and functional properties rather
than aesthetic and decorative characteristics.
a. 2 only
 They find application not only in clothing but
b. 1 and 2 only also in areas like agriculture, medical,
c. 2 and 3 only infrastructure, automotive, aerospace,
sports, defence and packaging.
d. 1, 2 and 3
4. d
 The Animal Welfare Board of India (AWBI) is
Answers a statutory advisory body on Animal Welfare
1. c Laws and promotes animal welfare in the
country.
 Elephanta Caves near Mumbai is a UNESCO
World Heritage site.  It was established in 1962 under Section 4 of
the Prevention of Cruelty to Animals
 The Elephanta Caves are located in Western Act, 1960.
India on Elephanta Island (also known as
the Island of Gharapuri (City of caves)).

www.shankariasacademy.com | www.iasparliament.com
 Its headquarters was shifted from Chennai,  It will significantly improve the financial
Tamilnadu to Ballabgarh in Faridabad resources and powers of the autonomous
district of Haryana in 2018. districts councils in Assam, Meghalaya,
Mizoram and Tripura, fulfilling
 With several objections raised against his longstanding aspirations of the tribal
functioning by Union Minister Maneka population in these North-eastern states.
Gandhi and several animal rights activists as
the backdrop, retired IAS officer SP Gupta has 9. d
resigned from the chairmanship of AWBI.
 The two-day National Chilika Bird festival
5. d includes photo exhibition seminars and visit
to Nalbana island inside Chilika lake.
 SFIO is a statutory corporate fraud
investigating agency in India. 10. c
 Initially, it was set up by a resolution adopted  The Karnataka Forest Department has issued
by the Government of India in 2003. permission to expand a private mini-hydel
project in the eco-sensitive zone (ESZ) of
 Later, Section 211 of the Companies Act, 2013, the Cauvery Wildlife Sanctuary, located
accorded the statutory status to the SFIO. in Karnataka, though it was rejected by the
 SFIO is a multi-disciplinary organization State Board for Wildlife twice.
under Ministry of Corporate Affairs.  Cauvery North Wildlife Sanctuary is
6. b located in Tamilnadu.

 Durgama Anchalare Malaria Nirakaran  Patrolling in the Satkosia Tiger


(DAMaN), is an initiative Reserve, Odisha is set to be strengthened as
of Odisha government. two trained elephants would be deployed there
shortly.
 It aims to deliver services to the most
inaccessible and hardest hit people of the  The two elephants are being brought from the
State. Similipal Tiger Reserve, Odisha.

 Through its DAMaN initiative, Odisha has  Trained elephants will help ground-level forest
emerged as an inspiration in the global fight guards‘ patrol deep in the forest where jeeps
against malaria. cannot go.
7. d
 The 2nd National Chilika Bird Festival began 29-01-2019
recently at Mangalajodi, with the aim to 1) Consider the following statements with
promote eco-tourism and preservation of birds
respect to National Agricultural Higher
in Odisha.
Education Project (NAHEP)
 The two-day festival, jointly organized 1. It aims to support participating
by Odisha Tourism and Chilika Agricultural Universities and ICAR to
Development Authority, includes photo provide more relevant and higher
exhibition seminars and visit to Nalbana quality education to Agricultural
island inside Chilika. University students.
8. d 2. It will be funded by the World Bank and
 The Union Cabinet has recently approved the the Indian Government on a 50:50
constitutional amendment which seeks to basis.
increase the powers of the autonomous council 3. It was launched by the Ministry of
recently. Human Resources and Development.
 The amendment to Article 280 and Sixth Which of the statement(s) given above is/are
Schedule of the Constitution has been correct?
approved.
a. 1 only

www.shankariasacademy.com | www.iasparliament.com
b. 1 and 2 only 2. It was introduced by the Organisation
for Economic Co-operation and
c. 2 and 3 only
Development (OECD).
d. 1, 2 and 3
3. Recently, India has signed an agreement
confirming its participation in PISA for
the first time.
2) Consider the following statements with
respect to Student READY scheme Which of the statement(s) given above is/are
correct?
1. Under this scheme, practical experience
of agriculture and entrepreneurship is a. 2 only
provided to undergraduate students.
b. 1 and 2 only
2. It was launched by the Indian Council of
c. 1 and 3 only
Agricultural Research (ICAR).
d. 1, 2 and 3
Which of the statement(s) given above is/are
correct?
a. 1 only 5) Consider the following statements with
respect to Sarus Crane
b. 2 only
1. It is the world‟s tallest flying bird,
c. Both 1 and 2
endemic to Indian sub-continent.
d. Neither 1 nor 2
2. It has been listed as Vulnerable under
the IUCN Red list.
3) Consider the following statements with 3. It is omnivorous, feeding on fish and
respect to Consultative Committees insects, as well as roots and plants.
1. These committees are constituted by the Which of the statement(s) given above is/are
Ministry of Parliamentary Affairs. correct?
2. Meetings of these Committees are held a. 2 only
both during the session and inter-
b. 1 and 3 only
session period of Parliament.
c. 2 and 3 only
3. These committees shall stand dissolved
upon dissolution of every Lok Sabha. d. 1, 2 and 3
Which of the statements given above are
correct?
6) HSPD-6 and Global Entry
a. 1 and 2 only Programme (GEP) are the initiative of which of
the following countries?
b. 1 and 3 only
a. U.S.A
c. 2 and 3 only
b. France
d. 1, 2 and 3
c. U.K
d. Canada
4) Consider the following statements with
respect to Programme for International
Student Assessment (PISA)
7) Consider the following statements with
1. It is a biennial international survey respect to Charter of Patient‟s Rights
which aims to evaluate the education
1. It has been developed by the Ministry of
system worldwide by testing the skills
Health and Family Welfare.
and knowledge of 15-year-old students.

www.shankariasacademy.com | www.iasparliament.com
2. India has recently adopted this charter a. 1-C; 2-D; 3-A; 4-B
which addresses violations of patient
b. 1-B; 2-D; 3-C; 4-A
rights.
c. 1-C; 2-D; 3-B; 4-A
Which of the statement(s) given above is/are
correct? d. 1-B; 2-C; 3-A; 4-D
a. 1 only
b. 2 only Answers
c. Both 1 and 2 1. b
d. Neither 1 nor 2  The Indian Council of Agricultural
Research (ICAR) has recently launched
Rs.1,100 crore ambitious National Agricultural
8) Consider the following statements Higher Education Project (NAHEP) to attract
1. India can make an extradition request talent and strengthen higher agricultural
to any country, even without having a education in the country.
bilateral extradition treaty with a  This project will be funded by the World
specific country. Bank and the Indian Government on a
2. Currently, India and Australia does not 50:50 basis.
have any extradition treaty between
 The objective of the Project is to support
them. Participating Agricultural Universities and
Which of the statement(s) given above is/are ICAR in providing more relevant and higher
correct? quality education to Agricultural University
students.
a. 1 only
 NAHEP addresses quality by supporting
b. 2 only
interested AUs to propose and implement
c. Both 1 and 2 technically sound and verifiable investments
that increase faculty performance, attract
d. Neither 1 nor 2 better students to these AUs, improve student
learning outcomes and raise their prospects for
future employability, particularly in the private
9) ―Zearalenone‖ sometimes seen in the news sector.
recently, is a
2. c
a. Fungal toxin
 The Student READY (Rural Entrepreneurship
b. Drug resistant bacteria Awareness Development Yojana) programme
c. Country in Western Africa is an initiative of Indian Council of
Agricultural Research (ICAR) to reorient
d. None of the above graduates of Agriculture and allied subjects for
ensuring and assuring employability and
develop entrepreneurs for emerging
10) Match the following knowledge intensive agriculture.
Folk Dance – State  It aims to provide rural entrepreneurship
1. Bhangra – a. Gujarat awareness, practical experience in real-life
situation in rural agriculture and creating
2. Karakkatam – b. Maharashtra awareness to undergraduate
3. Misra Raas – c. Punjab students about practical agriculture and
allied sciences.
4. Taakala – d. TamilNadu
3. d
Select the correct answer using the code given
below  Consultative committees are attached to
various ministries/departments of the Central

www.shankariasacademy.com | www.iasparliament.com
Government and consist of members of both  In the Indian subcontinent, it is found
the Houses of Parliament. in northern and central India, Terai
Nepal and Pakistan.
 The Minister / Minister of State in charge of
the Ministry concerned acts as the chairman of  It is the official state bird of Uttar Pradesh.
the consultative committee of that ministry.
 It is not only the tallest flying bird in the world,
 These committees provide a forum for it is also India‘s only resident breeding
informal discussions between the ministers crane, as per the Wildlife Trust of India
and the members of Parliament on policies (WTI).
and programmes of the government and the
manner of their implementation.  The International Union for Conservation of
Nature (IUCN) has marked it as ‗vulnerable‘
 These committees are constituted by in its list of threatened species.
the Ministry of Parliamentary Affairs.
 It is currently listed in the Schedule IV of the
 The Ministry also makes arrangements for Wildlife (Protection) Act, 1972.
holding their meetings both during the
session and the intersession period of  The population of the Sarus crane, a bird
Parliament. distinguishable by its red upper neck and
white collar, has climbed as per the 2018
 These committees are normally constituted census (summer).
after the new Lok Sabha is constituted, after
General Elections for the Lok Sabha. 6. a

 In other words, these committees shall  Homeland Security Presidential Directive


stand dissolved upon dissolution of (HSPD-6) is an agreement for exchange of
every Lok Sabha and shall be reconstituted terrorist screening information between the
upon constitution of each Lok Sabha. Terrorist Screening Centre (TSC) of the U.S.
and the partner country‘s security agency.
4. a
 Global Entry is a US Customs and Border
 PISA — introduced in 2000 by OECD — tests Protection (CBP) program that allows speedy
the learning levels of 15-year-olds in clearance for low-risk travellers upon their
reading, mathematics and science. arrival in the US.
 The test is conducted every three years. 7. d
 India stayed away from PISA in 2012 and 2015  Charter of Patient‘s Rights has been developed
on account of its dismal performance in 2009, by National Human Rights Commission
when it was placed 72nd among the 74 (NHRC).
participating countries.
 In August 2018, the Ministry of Health and
 Confirming India’s participation in the Family Welfare announced its plans to
Programme for International Student implement this Charter.
Assessment (PISA) in 2021, the HRD Ministry
has signed an agreement with the  Although nearly 4 months have elapsed since
Organisation for Economic Cooperation this declaration of intent, there seems to have
Development (OECD) recently. been no further progress on finalisation
and adoption of the charter.
5. c
8. a
 The Sarus crane is the tallest flying bird in
the world.  According to the website of the Ministry of
External Affairs, India can make
 It has three disjunct populations in an extradition request to any country.
the Indian sub-continent, south-east
Asia and northern Australia.  While India‘s treaty partners have treaty
obligations to consider India‘s requests, in the
absence of a treaty, it is a matter for the foreign
country to consider, in accordance with its

www.shankariasacademy.com | www.iasparliament.com
domestic laws and procedures, whether the  Vice President has recently witnesses folk
country can agree to India‘s extradition dances performed by Tableaux Artists.
request on the basis of an assurance of
reciprocity.  Artists from Tamil Nadu presented
the Karakkatam folk dance, while Taakala
 India has extradition treaties with 43 countries Folk Dance was performed by artists from
and extradition arrangements with 10 Maharashtra.
countries.
 North Eastern states Arunachal Pradesh,
 India and Australia signed an extradition Assam, Sikkim and Tripura presented Monpa,
treaty in 2008, but it came into force only in Satoiya Nitya, Tamang Seto and Mamita folk
2011. dances.
 Under this treaty, both countries decided to  Artists from Jammu Kashmir performed the
recognise extraditable offences which are Fusim Dance reflecting the compositie culture
punishable by an imprisonment of at least one of the State.
year or severe penalty.
 The artistes from Gujarat performed Misra
 India is all set to sign a revamped Raas, while Hudka Chhudka from Uttarakhand
extradition treaty with Australia that and Bhangra from Punjab were also
would enable both countries to exchange performed.
information on spies, terrorism and human
traffickers.
 The extradition treaty signed with 30-01-2019
Australia earlier was general in 1) Consider the following statements with
nature. This one will be more crime respect to Junk Bonds
specific. It will focus on narcotics, human
trafficking, spies and terrorism. 1. These bonds are highly rated corporate
bonds.
9. a
2. These bonds are issued by low rated
 Zearalenone is a fungal toxin infesting companies.
cereals such as wheat, maize and barley.
3. In India, Junk bonds are regulated by
 It attacks crops while they are growing, but can SEBI.
also develop when cereals are stored without
being dried fully. Which of the statement(s) given above is/are
correct?
 Recently, a Journal of Food Science study
a. 1 only
detected zearalenone in wheat, rice, corn
and oats from markets in Uttar Pradesh. b. 2 only
 The Food Safety and Standards Authority of c. 2 and 3 only
India does not impose maximum limits
d. 1, 2 and 3
for zearalenone, though the European
Union (EU) does.
 Zearalenone behaves like oestrogen, the 2) Consider the following statements with
female sex hormone, and could cause respect to D – Voters
endocrine disturbances in humans.
1. D – Voters are those who have been
 Some studies over the past two decades have disenfranchised for their alleged
suggested that girls exposed to zearalenone acquisition of the citizenship of another
might exhibit early puberty, while others have country.
linked elevated blood levels of zearalenone to 2. They have to prove their citizenship in
mammary gland and prostate tumours. the Foreigners tribunal in order to get
10. a back their voting right.

www.shankariasacademy.com | www.iasparliament.com
Which of the statement(s) given above is/are a. 1 only
correct?
b. 2 only
a. 1 only
c. Both 1 and 2
b. 2 only
d. Neither 1 nor 2
c. Both 1 and 2
d. Neither 1 nor 2
6) Consider the following statements with
respect to Symbol allotment by the Election
Commission of India (ECI)
3) Consider the following statements with
respect to National Statistical Commission 1. The Election commission has three
(NSC) categories of symbols namely, Reserved,
Free and Open category.
1. It is a statutory body established under
the National Statistical Commission Act, 2. Reserved symbols are allotted to
2005. candidates sponsored by recognised
state or national parties.
2. Niti Aayog CEO is an ex-officio member
of NSC. 3. Open symbols are allotted to candidates
sponsored by a party, which secures six
3. It works under the Ministry of Statistics
percent of valid votes pollen in any four
and Programme Implementation
or more states at a general election to
(MoSPI).
the Lok Sabha.
Which of the statement(s) given above is/are
Which of the statement(s) given above is/are
correct?
correct?
a. 1 only
a. 2 only
b. 1 and 3 only
b. 1 and 2 only
c. 2 and 3 only
c. 1 and 3 only
d. 1, 2 and 3
d. 1, 2 and 3

4) ―Chin Refugees‖ sometimes seen in the news


7) Which of the following countries
recently, migrated to India from which of the
is/are not the members of Comprehensive and
following neighbouring countries?
Progressive Agreement for Trans-Pacific
a. China Partnership (CPTPP)?
b. Bangladesh 1. U.S.A
c. Myanmar 2. Canada
d. Afghanistan 3. China
4. India
5) Consider the following statements with 5. Japan
respect to Golden Langur Conservation Project
6. Australia
1. It has been funded by the Central Zoo
Select the correct answer using the code given
Authority (CZA).
below
2. Golden langur, founded only in Assam
a. 1 and 4 only
and Bhutan is listed as endangered
under the IUCN Red list. b. 1, 3 and 4 only
Which of the statement(s) given above is/are c. 2, 5 and 6 only
correct?
d. 1, 2, 3 and 4 only

www.shankariasacademy.com | www.iasparliament.com
 Junk bonds are a high-yielding debt
instrument without an investment grade
8) ―Pong Dam wetlands‖ sometimes seen in
(noninvestment-grade bond).
the news recently, is located in which of the
following states?  These bonds carry a credit rating of BB and
a. Gujarat lower, because they have a higher risk of
default.
b. Jammu and Kashmir
 Typically, these bonds also give higher
c. Uttarakhand returns as opposed to investment grade
d. Himachal Pradesh bonds, making them attractive to investors.
 The Securities and Exchange Board of India
(SEBI) allowed companies to raise funds by
9) Consider the following statements with issuing junk bonds or those below investment
respect to Amur Tigers grade in 2007.
1. These are the largest and heaviest of all 2. b
cats with the thickest fur.
 D-voter or Doubtful voter is a category of
2. The main difference between the voters in Assam.
Siberian tiger and the Amur Tiger is its
migratory pattern.  In 1997, during intensive revision of electoral
rolls, the Election Commission had ordered
Which of the statement(s) given above is/are that the letter ‗D‘ be written against names of
correct? those voters who failed to provide proof of
a. 1 only their citizenship.
b. 2 only  Cases of all those were referred to
the foreigners‘ tribunals (FT) and they
c. Both 1 and 2 have to prove their citizenship in the tribunal.
d. Neither 1 nor 2
 If they fail to prove their citizenship, the FT
designates them Declared Foreigners (DF) and
they are sent to Detention Camps.
10) Consider the following with respect to
Hansen‟s disease  The ‗D‘ voters are not allowed to cast their
votes.
1. It is a bacterial disease.
3. c
2. It is not hereditary and completely
curable disease.  The Government of India through a resolution
in 2005 set up the National Statistical
Which of the statement(s) given above is/are
Commission (NSC).
correct?
a. 1 only  The setting up of the NSC followed the
decision of the Cabinet to accept the
b. 2 only recommendations of the Rangarajan
c. Both 1 and 2 Commission, which reviewed the Indian
Statistical System in 2001.
d. Neither 1 nor 2
 The Commission consists of a part-time
Chairperson, four part-time Members
and CEO, Niti Aayog, as an ex-officio
Answers Member. The Chief Statistician of India (CSI)
1. c is the Secretary to the Commission.
 According to SEBI, Junk bonds are high  The chairman and an external member of the
yield bonds issued by low rated National Statistical Commission (NSC) have
companies. resigned recently, expressing disappointment

www.shankariasacademy.com | www.iasparliament.com
over the treatment being meted out to the NSC assembly, it will be recognised as a national
by the government. party.
4. c 7. b
 In November 2017, about 1,600 Chins, many  The Comprehensive and Progressive
of whom were women and children, fled to Agreement for Trans-Pacific Partnership
Lawngtlai district of Mizoram following a (CPTPP) is a free trade agreement involving 11
military offensive against the Arakan Army countries in the Asia-Pacific region, including
(AA) militants in the Chin State of New Zealand, Australia, Brunei
Myanmar. Darussalam, Canada, Chile, Japan,
Malaysia, Mexico, Peru, Singapore, and Viet
 Eight organisations of the Chakma community Nam.
had recently submitted a memorandum to the
Ministry of Home Affairs seeking the inclusion 8. d
of Chin refugees in India by further amending
 The bird watchers in association with the state
the Citizenship (Amendment) Bill, 2016.
Forest Department are organizing the annual
5. c census of waterfowl species at the famed Pong
wetlands of Kangra Valley, Himachel
 Golden langur is a charismatic primate species Pradesh.
conspicuous by its lustrous creamy-golden fur
and found only in few forest patches of Assam  Water fowls are the birds that depend on water
and Bhutan. bodies for roosting and feeding.
 The golden langur ( Trachypithecus geei ) is  It is the only place in the country after the
currently been listed as endangered under Bharatpur sanctuary in Rajasthan where the
the IUCN Red list. red-necked grebe descends every year.
 The golden langur conservation project was  The wetland is one of the largest man- made
undertaken at the Assam State Zoo in wetlands of the Northern India.
Guwahati during the 2011-12 fiscal.
 It is formed by the construction of Pong Dam
 It has been funded by the Central Zoo during 1974 across the Beas River.
Authority.
 The Pong dam reservoir is also known Maha
 Assam Environment and Forest Ministry has Rana Pratap Sagar.
recently announced the success of the Golden
Langur Conservation Breeding Programme in  It was declared a ―RAMSAR SITE‖ in the year
the State. 2002.

6. a 9. a

 The EC has two lists of symbols: reserved  Amur tigers, which were earlier called
and free. Siberian tigers, are listed as
an endangered species.
 While reserved symbols are allotted to
candidates sponsored by recognised  Amur tigers were once found across the
state or national parties, a list of free Russian Far East, northern China and the
symbols is prepared for independent Korean peninsula.
candidates or those affiliated to unrecognised  At present, Amur Tigers are mostly found in
parties. Amur Valley.
 According to EC, the second category of  Amur tigers are the largest and heaviest of all
candidates has to choose three symbols from cats with the thickest fur.
the list of free symbols, which are allotted
during nomination.  In December 2018, 11 Amur tigers were
spotted in the upper reaches of the
 If a party secures six per cent of valid votes Dibang Valley in Arunachal Pradesh.
polled in any four or more states at a general
election to the Lok Sabha or to the legislative 10. c

www.shankariasacademy.com | www.iasparliament.com
 Leprosy, also known as Hansen‘s disease, is 2. It always exists near the poles, but
a bacterial disease. weakens in summer and strengthens in
winter.
 It affects skin and nerves which can lead to
physical deformity and disability if left Which of the statement(s) given above is/are
untreated. correct?

 It is not hereditary and completely a. 1 only


curable, as opposite to general public views b. 2 only
on leprosy.
c. Both 1 and 2
 It is only mildly infectious (i.e) more than 85%
of cases are non-infectious and over 95% of the d. Neither 1 nor 2
population has a natural immunity to the
disease.
4) Consider the following statements with
respect to Aero India Show
31-01-2019 1. It is an annual exhibition show
organised by the Ministry of Defence.
1) Consider the following statements with
respect to Department for Promotion of 2. The tagline for this year‟s event is “The
Industry and Internal Trade (DPIIT) Runway to a Billion Opportunities”.
1. It functions under the Ministry of 3. Drone Olympics, scheduled to be held in
Consumer Affairs. India for the first time is a part of the
Aero India 2019 event.
2. While Department of Industrial Policy
and Promotion (DIPP) is responsible for Which of the statement(s) given above is/are
external trade, DPIIT is responsible for correct?
Internal Trade in India.
a. 2 only
Which of the statement(s) given above is/are
correct? b. 1 and 2 only

a. 1 only c. 2 and 3 only

b. 2 only d. 1, 2 and 3

c. Both 1 and 2
d. Neither 1 nor 2 5) ―Beveridge Curve‖ in economics, shows the
relationship between?
a. Income inequality and wealth distribution
2) “Corruption Perceptions Index 2018” was
released recently by which of the following? b. Unemployment and Job vacancy

a. United Nations Office on Drugs and Crime c. Average tax rate and total tax revenue

b. Organisation for Economic Co-operation and d. Inflation and unemployment


Development
c. International Anti-Corruption Academy 6) ―Future of Rail Report‖ which was released
d. Transparency International recently, was prepared by which of the
following?
a. Transport standards organisation
3) Consider the following statements with
respect to Polar Vortex b. Ministry of Indian Railways

1. It is a large area of low pressure that c. International Energy Agency


consistently overlies the North and d. None of the above
South poles.

www.shankariasacademy.com | www.iasparliament.com
c. Both 1 and 2
7) Consider the following statements with d. Neither 1 nor 2
respect to Sahitya Akademi Award
1. It is being conferred annually for
10) Beypore port is located in which of the
authors of works of outstanding literary
following states?
merit in Indian languages.
a. Kerala
2. Awards are currently being given only in
22 scheduled languages of India. b. Gujarat
Which of the statement(s) given above is/are c. Andhra Pradesh
correct?
d. Odisha
a. 1 only
b. 2 only
Answers
c. Both 1 and 2
1. d
d. Neither 1 nor 2
 The government has recently renamed the
Department of Industrial Policy and
8) Consider the following with respect to Promotion (DIPP) as the Department for
International Energy Agency (IEA) Promotion of Industry and Internal Trade
(DPIIT).
1. It is an inter-governmental
organisation.  DPIIT will work under the Ministry of
Commerce and Industry.
2. India is one of the member countries of
IEA.  In addition to the previous responsibilities of
the erstwhile DIPP, DPIIT will also look in to
3. World Energy Outlook (WEO) is being
matters related to promotion of internal trade,
published by the IEA. including retail trade, welfare of traders and
Which of the statement(s) given above is/are their employees, facilitating ease of doing
correct? business and start-ups.
a. 3 only  Earlier issues related to internal trade was
looked after by the consumer affairs
b. 1 and 3 only
ministry.
c. 2 and 3 only
2. d
d. 1, 2 and 3
 ―Corruption Perceptions Index 2018‖,
the Transparency International‘s latest
report on business leaders‘ perceptions of
9) Consider the following statements with
corruption, had India marginally improve its
respect to Southern Lights
score to 41 (from 40 in 2017), and its position
1. It is the southern counterpart of aurora to 78 out of 180 countries in 2018 against
borealis. 2017‘s 81st, where it had slid from 79 in 2016.
2. It is caused by the collision between 3. c
fast-moving electrons from space with
 The Polar vortex is a large area of low
the oxygen and nitrogen in Earth‟s
pressure that consistently overlies the North
upper atmosphere.
and South poles.
Which of the statement(s) given above is/are
correct?  It always exists near the poles, but weakens
in summer and strengthens in winter.
a. 1 only
b. 2 only

www.shankariasacademy.com | www.iasparliament.com
 The term "vortex" refers to the counter- (IEA), was released recently by the Ministry of
clockwise flow of air that helps keep the colder Railways, India.
air near the Poles.
 It analyses the current and future importance
 Usually this mass of cold polar air remains of rail around the world through the
over the poles - strong and compact. perspective of its energy and environmental
implications.
 But sometimes, during winter in the northern
hemisphere, the vortex becomes weak and  This first ever global report has a focus on
expands, sending Arctic cold air southward. India, elaborating on the unique social and
economic role of rail in India, together with its
 Parts of the weak vortex will continue to creep great enduring potential, to show how India
away from the Arctic region, influencing the can extend and update its networks to harness
path of the jet stream (an area of fast-moving rail at a scope and scale that is unparalleled.
air high in the atmosphere).
7. a
 The polar vortex can lead to major cold air
outbreaks in the Northern Hemisphere,  Every year since its inception in 1954, the
including North America, Europe and Asia. Sahitya Akademi Award prizes to the most
outstanding books of literary merit published
4. c in any of the major Indian languages
 Aero India Exhibition show is being recognised by the Akademi.
organised every two years.  Awards are currently being given for 24
 The tagline for Aero India 2019 is ―The languages (22 Scheduled Languages and
Runway to a Billion Opportunities‖. English and Rajasthani).

 Government of India is organizing a Drone 8. b


Olympics in the Aero India 2019 event.  The IEA is an inter-governmental
 Being the first of such kind global event on organisation that works to ensure reliable,
drones, it will be a great opportunity for global affordable and clean energy for its 30 member
and Indian drone manufacturers to showcase countries and 8 association countries.
their Unmanned Aerial Vehicles (UAVs).  India is one of the 8 association countries (not
5. b the member of IEA)

 Beveridge curve refers to a graphical 9. c


representation that shows the relationship  The aurora borealis and aurora
between the unemployment rate (on the australis – often called the northern
horizontal axis) and the job vacancy lights and southern lights respectively –
rate (on the vertical axis) in an economy. are common occurrences at high northern and
 The Beveridge curve usually slopes southern latitudes.
downwards because times when there is  The typical aurora is caused by collisions
high job vacancy in an economy are also between fast-moving electrons from
marked by relatively low unemployment since space with the oxygen and nitrogen in
companies may actually be actively looking to Earth‘s upper atmosphere.
hire new people.
 During one kind of solar storm called
 By the same logic, a low job vacancy rate a coronal mass ejection, the sun burps out
usually corresponds with high unemployment a huge bubble of electrified gas that can travel
as companies may not be looking to hire many through space at high speeds.
people in new jobs.
 When a solar storm comes toward earth, some
6. c of the energy and small particles can travel
 ―The Future of Rail‖ the first-of-a-kind down the magnetic field lines at the north and
report by International Energy Agency south poles into Earth‘s atmosphere.

www.shankariasacademy.com | www.iasparliament.com
 There, the particles interact with gases in
Earth‘s atmosphere resulting in beautiful
displays of light in the sky.
 Oxygen gives off green and red light. Nitrogen
glows blue and purple.
10. a
 With the Customs Department‘s decision to
introduce Electronic Data Interface (EDI)
clearance facility, the Beypore port in
Kerala will soon resume its international
container shipping operations, inviting local
importers who were heavily dependent on
other ports for their business operations till
now.

www.shankariasacademy.com | www.iasparliament.com

Vous aimerez peut-être aussi